R4-1 Flashcards

1
Q

Smith made a gift of property to Thompson. Smith’s basis in the property was $1,200. The fair market value at the time of the gift was $1,400. Thompson sold the property for $2,500. What was the amount of Thompson’s gain on the disposition?

a.

$0

b.

$1,100

c.

$1,300

d.

$2,500

A

Choice “c” is correct. The general rule for the basis on gifted property is that the donee receives the property with a rollover cost basis (equal to the donor’s basis). An exception exists where the fair market value of the property at the time of the gift is less than the donor’s basis. That is not the case in this question; thus, the calculation of the gain on the disposition of the property is:

Amount realized $ 2,500

Basis (1,200)

Gain recognized $ 1,300

Choice “a” is incorrect. This choice could be correct if the facts of the question met the exception whereby no gain or loss is recognized when a donee sells gifted property for an amount between the donor’s basis and the fair market value at the date of the gift.

Choice “b” is incorrect. This choice uses the basis as the fair market value of the property. Fair market value of property at date of death is used as the basis for inherited property (in all years except 2010), not gifted property.

Choice “d” is incorrect. This choice assumes that Thompson’s basis is zero. His basis is $1,200 as indicated above.

How well did you know this?
1
Not at all
2
3
4
5
Perfectly
2
Q

Leker exchanged a van that was used exclusively for business and had an adjusted tax basis of $20,000 for a new van. The new van had a fair market value of $10,000, and Leker also received $3,000 in cash. What was Leker’s tax basis in the acquired van?

a.

$13,000

b.

$20,000

c.

$7,000

d.

$17,000

A

Choice “d” is correct. $17,000 is the tax basis in the van.

The basis for like-kind exchanges is computed as follows:

Basis of old property $ 20,000

Less: Boot received (3,000)

New basis $ 17,000

Alternate calculation: FMV of new van $10,000 + deferred loss $7,000 = New basis $17,000.

The general rule is the gain is recognized to the extent boot is received. As the transaction results in a loss to Leker (he received an asset worth $10,000 plus $3,000 cash less a $20,000 tax basis equals $7,000 loss) no gain is recognized and the $3,000 received reduces his basis in the new asset.

Choice “b” is incorrect. Basis must be reduced by non-like-kind assets (boot) received.

Choice “a” is incorrect. For non-like-kind exchanges, the basis would be the FMV of the assets received ($10,000 FMV plus $3,000 Boot). However, because both assets have similar use, this is a like-kind exchange, which follows the rule above.

Choice “c” is incorrect. The basis of the old property is used to calculate the basis of the new property, less any boot received.

How well did you know this?
1
Not at all
2
3
4
5
Perfectly
3
Q

Capital assets include:

a.

Seven-year MACRS property used in a corporation’s trade or business.

b.

A manufacturing company’s investment in U.S. Treasury bonds.

c.

A corporation’s accounts receivable from the sale of its inventory.

d.

A corporate real estate developer’s unimproved land that is to be subdivided to build homes, which will be sold to customers.

A

Choice “b” is correct. Investment assets of a taxpayer that are not inventory are capital assets. The manufacturing company would have capital assets including an investment in U.S. Treasury bonds.

Choice “c” is incorrect. Accounts receivable generated from the sale of inventory are excluded from the statutory definition of capital assets.

Choice “a” is incorrect. Depreciable property used in a trade or business is excluded from the statutory definition of capital assets.

Choice “d” is incorrect. Land is usually a capital asset, but when it is effectively inventory, as when it is used by a developer to be subdivided, it is excluded from the statutory definition of capital assets.

How well did you know this?
1
Not at all
2
3
4
5
Perfectly
4
Q

Conner purchased 300 shares of Zinco stock for $30,000, 20 years ago. On May 23 of the current year, Conner sold all the stock to his daughter Alice for $20,000, its then fair market value. Conner realized no other gain or loss during the year. On July 26 of the current year, Alice sold the 300 shares of Zinco for $25,000.

What amount of the loss from the sale of Zinco stock can Conner deduct in the current year?

a.

$10,000

b.

$5,000

c.

$3,000

d.

$0

A

Choice “d” is correct. Even though Conner has a realized loss of $10,000 on this transaction he cannot deduct the loss since it was incurred in a transaction with his daughter, a related party.

Choice “c” is incorrect. $3,000 is the limit on deductible net capital losses. However, Conner cannot deduct this loss, since it was incurred in a transaction with his daughter, a related party.

Choice “b” is incorrect. Conner’s realized loss on the sale is $10,000 ($20,000 proceeds less $30,000 basis). However, Conner cannot deduct this loss, since it was incurred in a transaction with his daughter, a related party.

Choice “a” is incorrect. $10,000 is Conner’s realized loss on the sale. However, Conner cannot deduct this loss, since it was incurred in a transaction with his daughter, a related party.

How well did you know this?
1
Not at all
2
3
4
5
Perfectly
5
Q

Conner purchased 300 shares of Zinco stock for $30,000, 20 years ago. On May 23 of the current year, Conner sold all the stock to his daughter Alice for $20,000, its then fair market value. Conner realized no other gain or loss during the year. On July 26 of the current year, Alice sold the 300 shares of Zinco for $25,000.

What was Alice’s recognized gain or loss on her sale?

a.

$5,000 long-term gain.

b.

$0

c.

$5,000 long-term loss.

d.

$5,000 short-term loss.

A

Choice “b” is correct. Alice has a realized gain of $5,000 on the transaction: $25,000 sales price less $20,000 purchase price. However, she can reduce the gain, but not below zero, by the amount of loss her father could not deduct on the sale to her. Thus, Alice can reduce her gain by up to $10,000, but not below zero. Here, the gain is $5,000, so it is reduced to zero. Conner should have sold the stock in the open market so that he could deduct the entire loss. Alice could then have purchased the stock in the open market.

Choice “a” is incorrect. $5,000 is Alice’s realized long-term gain on the sale. However, she can reduce the gain, but not below zero, by the amount of loss her father could not deduct on the sale to her.

Choice “d” is incorrect. Alice has a realized gain of $5,000 on the sale. However, since she is related to Conner, her holding period includes his holding period. Therefore, her realized gain is long-term. In addition, she can reduce the gain, but not below zero, by the amount of loss her father could not deduct on the sale to her.

Choice “c” is incorrect. Alice can reduce the gain by the amount of loss her father could not deduct on the sale to her. However, she cannot reduce the gain below zero.

How well did you know this?
1
Not at all
2
3
4
5
Perfectly
6
Q

If the executor of a decedent’s estate elects the alternate valuation date and none of the property included in the gross estate has been sold or distributed, the estate assets must be valued as of how many months after the decedent’s death?

a.

3

b.

12

c.

9

d.

6

A

Choice “d” is correct.

Rule: The executor can elect to use an alternate valuation date rather than the decedent’s date of death to value the property included in the gross estate. The alternate date is generally six months after the decedent’s death or the earlier date of sale or distribution.

Note: The valuation of the assets in an estate impacts the recipient as basis of the inherited assets.

Choices “b”, “c”, and “a” are incorrect, per the above rule.

How well did you know this?
1
Not at all
2
3
4
5
Perfectly
7
Q

In December, Year 10, Davis, a single taxpayer, purchased a new residence for $200,000. Davis lived in the new residence continuously from Year 10 until selling the new residence in July, Year 17 for $455,000. What amount of gain is recognized from the sale of the residence on Davis’ Year 17 tax return?

a.

$455,000

b.

$255,000

c.

$5,000

d.

$0

A

Choice “c” is correct. Provided Davis has lived in his home for a total of 2 years out of the 5 years preceding his sale of his residence, as a single taxpayer he may exclude up to $250,000 of gain on its sale. The basis on the residence sold in Year 17 is equal to its cost ($200,000).

Selling Price $ 455,000

Less: Basis (200,000)

Realized Gain 255,000

Less: Excluded Amount (250,000)

Recognized Gain $ 5,000

Choices “a”, “b”, and “d” are incorrect, per the above.

How well did you know this?
1
Not at all
2
3
4
5
Perfectly
8
Q

Smith, an individual calendar-year taxpayer, purchased 100 shares of Core Co. common stock for $15,000 on December 15, Year 1, and an additional 100 shares for $13,000 on December 30, Year 1. On January 3, Year 2, Smith sold the shares purchased on December 15, Year 1, for $13,000. What amount of loss from the sale of Core’s stock is deductible on Smith’s Year 1 and Year 2 income tax returns?

~Year 1
~Year 2
a.

$2,000

$0

b.

$0

$2,000

c.

$0

$0

d.

$1,000

$1,000

A

Choice “c” is correct. In Year 1, no sale of stock occurred so there would be no loss. In Year 2, there is a $2,000 loss realized ($15,000 basis less $13,000 received), but it is not deductible because it is a wash sale. A wash sale occurs when a taxpayer sells stock at a loss and invests in substantially identical stock within 30 days before or after the sale. In this case, Smith reinvested in an additional 100 shares four days prior to selling 100 shares of the same stock at a loss. The $2,000 disallowed loss would, however, increase the basis of the new shares by $2,000.

Choice “b” is incorrect. The $2,000 loss realized in Year 2 is disallowed under the wash sale rules.

Choice “d” is incorrect. In Year 1, there is no loss since no shares were sold. In Year 2, the $2,000 loss is disallowed under the wash sale rules.

Choice “a” is incorrect. In Year 1, there is no possible loss since no shares were sold.

How well did you know this?
1
Not at all
2
3
4
5
Perfectly
9
Q

In Year 3, Fay sold 100 shares of Gym Co. stock to her son, Martin, for $11,000. Fay had paid $15,000 for the stock in Year 1. Subsequently in Year 3, Martin sold the stock to an unrelated third party for $16,000.

What amount of gain from the sale of the stock to the third party should Martin report on his Year 3 income tax return?

a.

$5,000

b.

$0

c.

$4,000

d.

$1,000

A

Choice “d” is correct. Losses between related parties are disallowed. Therefore, Fay’s $4,000 capital loss ($15,000 basis less $11,000 received) is disallowed because she sold the stock to her son, a related party. When her son sells the stock to an unrelated party, however, he can use the $4,000 disallowed loss to reduce any gain he realized from the sale (but not to create or increase a loss). His realized gain is $5,000 ($16,000 received less $11,000 basis), but he can reduce it by $4,000 to $1,000 using his mother’s disallowed loss. Employing the “Pass Key” in the text, Martin sold the stock for higher than Fay purchased it. The donor’s basis (i.e., $15,000) is, therefore, used to determine gain on the sale by Martin.

Choice “b” is incorrect. Martin’s gain, after reducing it by his mother’s disallowed loss, is reported on his tax return.

Choice “c” is incorrect. The $4,000 disallowed loss to his mother reduces his $5,000 gain.

Choice “a” is incorrect. The $5,000 gain is reduced by his mother’s $4,000 disallowed loss.

How well did you know this?
1
Not at all
2
3
4
5
Perfectly
10
Q

Hall, a divorced person and custodian of her 12-year-old child, filed her Year 9 federal income tax return as head of a household. She submitted the following information to the CPA who prepared her Year 9 return:

In June, Year 9, Hall’s mother gifted her 100 shares of a listed stock. The donor’s basis for this stock, which she bought in Year 1, was $4,000, and market value on the date of the gift was $3,000. Hall sold this stock in July, Year 9 for $3,500. The donor paid no gift tax. What was Hall’s reportable gain or loss in Year 9 on the sale of the 100 shares of stock gifted to her?

a.

$0

b.

$1,000 loss.

c.

$500 loss.

d.

$500 gain.

A

Choice “a” is correct.

Rule: The basis of property received as a gift in the hands of the donee depends on whether the selling price of the property is more or less than the basis for gain or loss.

If the property is sold at a gain, the basis to the donee is the same as it would be in the hands of the donor. If the property is sold at a loss, the basis to the donee is the same as it would be in the hands of the donor or the FV of the property at the date of the gift, whichever is lower. In some cases, such as this fact situation, there is neither gain nor loss on the sale of the gift, because the selling price is less than the basis for gain and more than the basis for loss.

Choices “d”, “c”, and “b” are incorrect, per the above rule.

How well did you know this?
1
Not at all
2
3
4
5
Perfectly
11
Q

In a “like-kind” exchange of an investment asset for a similar asset that will also be held as an investment, no taxable gain or loss will be recognized on the transaction if both assets consist of:

a.

Convertible preferred stock.

b.

Rental real estate located in different states.

c.

Partnership interests.

d.

Convertible debentures.

A

Choice “b” is correct. No taxable gain or loss will be recognized on a like-kind exchange if both assets are tangible property. Rental real estate located in different states qualifies for a like-kind exchange.

Choices “d”, “a”, and “c” are incorrect. In order to meet the “like-kind exchange” requirements for nonrecognition of gain or loss, the property exchanged must be tangible property. Convertible debentures, convertible preferred stock, and partnership interests are not considered tangible property.

Exception: If the same class of stock of the same corporation is exchanged, it will qualify for “substituted basis.”

How well did you know this?
1
Not at all
2
3
4
5
Perfectly
12
Q

In Year 9, Joan Reed exchanged commercial real estate that she owned for other commercial real estate plus cash of $50,000. The following additional information pertains to this transaction:

Property given up by Reed

Fair value $ 500,000

Adjusted basis 300,000

Property received by Reed

Fair value 450,000

What amount of gain should be recognized in Reed’s Year 9 income tax return?

a.

$0

b.

$200,000

c.

$50,000

d.

$100,000

A

Choice “c” is correct. $50,000 is Reed’s recognized gain in Year 9.

Rule: Gain is only recognized on an exchange of “like-kind” property for the lesser of the amount of “gain realized” or the amount of “boot” received in the exchange.

Fair value of property received $ 450,000

Amount of cash (“boot”) received 50,000

Total amount realized $ 500,000

Basis of property given up (300,000)

Gain realized $ 200,000

Gain recognized* $ 50,000

* Gain recognized is the lesser of the amount of “gain realized” or amount of the “boot” received.

Choices “b”, “d”, and “a” are incorrect, per the above rule.

How well did you know this?
1
Not at all
2
3
4
5
Perfectly
13
Q

Platt owns land that is operated as a parking lot. A shed was erected on the lot for the related transactions with customers. With regard to capital assets and Section 1231 assets, how should these assets be classified?

~Land
~Shed
a.

Capital

Capital

b.

Section 1231

Capital

c.

Capital

Section 1231

d.

Section 1231

Section 1231

A

Choice “d” is correct. Because the parking lot and the shed constitute real estate and depreciable assets used in a trade or business, respectively, they are not capital assets per the definition below.

Note: The parking lot and shed will fall under Section 1231 (provided they are used in the business over 12 months) and possibly Section 1250 and 1245, respectively, upon sale of the assets.

Capital assets are defined as all property held by the taxpayer, except:

Property normally included in inventory or held for sale to customers in the ordinary course of business.

Depreciable property and real estate used in business.

Accounts and notes receivable arising from sales or services in the taxpayer’s business.

Copyrights, literary, musical, or artistic compositions held by the original artist. (Exception: Sales of musical compositions held by the original artist receive capital gain treatment.)

Treasury stock.

Choices “a”, “b”, and “c” are incorrect, per the above rule.

How well did you know this?
1
Not at all
2
3
4
5
Perfectly
14
Q

Lee qualified as head of a household for Year 9 tax purposes. Lee’s Year 9 taxable income was $100,000, exclusive of capital gains and losses. Lee had a net long-term loss of $8,000 in Year 9. What amount of this capital loss can Lee offset against Year 9 ordinary income?

a.

$0

b.

$8,000

c.

$4,000

d.

$3,000

A

Choice “d” is correct. The capital loss deduction is limited to $3,000 per year with the excess carried forward indefinitely. In this case, Lee can deduct $3,000 against his income and carry forward the remaining $5,000.

Choices “a”, “c”, and “b” are incorrect based on the above explanation.

How well did you know this?
1
Not at all
2
3
4
5
Perfectly
15
Q

Greller owns 100 shares of Arden Corp., a publicly traded company, which Greller purchased on January 1, Year 1, for $10,000. On January 1, Year 3, Arden declared a 2-for-1 stock split when the fair market value (FMV) of the stock was $120 per share. Immediately following the split, the FMV of Arden stock was $62 per share. On February 1, Year 3, Greller had his broker specifically sell the 100 shares of Arden stock received in the split when the FMV of the stock was $65 per share. What is the basis of the 100 shares of Arden sold?

a.

$6,500

b.

$6,200

c.

$6,000

d.

$5,000

A

Choice “d” is correct. The receipt of a nontaxable stock dividend will require the shareholder to spread the basis of his original share over both the original shares and the new shares received resulting in the same total basis, but a lower basis per share of stock held. Therefore, Greller’s total basis remains the same, $10,000, but is now split between 200 shares (a 2-for-1 split and he originally owned 100 shares). Therefore, his basis per share goes from $100/share ($10,000/100) to $50/share ($10,000/200). Consequently, his basis in 100 share is 100 x $50 = $5,000.

Choices “c”, “b”, and “a” are incorrect, per the above explanation.

How well did you know this?
1
Not at all
2
3
4
5
Perfectly
16
Q

Farr made a gift of stock to her child, Pat. At the date of gift, Farr’s stock basis was $10,000 and the stock’s fair market value was $15,000. No gift taxes were paid. What is Pat’s basis in the stock for computing gain?

a.

$0

b.

$15,000

c.

$10,000

d.

$5,000

A

Choice “c” is correct. Property acquired as a gift generally retains the rollover cost basis that it had in the hands of the donor at the time of the gift. Basis is increased by any gift tax paid that is attributable to the net appreciation in the value of the gift. Since there were no gift taxes paid, Pat’s basis for computing a gain is the rollover cost (basis), $10,000.

Choices “a”, “d”, and “b” are incorrect, per the explanation above.

How well did you know this?
1
Not at all
2
3
4
5
Perfectly
17
Q

Allen owns 100 shares of Prime Corp., a publicly traded company, which Allen purchased on January 1, Year 1, for $10,000. On January 1, Year 3, Prime declared a 2-for-1 stock split when the fair market value (FMV) of the stock was $120 per share. Immediately following the split, the FMV of Prime stock was $62 per share. On February 1, Year 3, Allen had his broker specifically sell the 100 shares of Prime stock received in the split when the FMV of the stock was $65 per share. What amount should Allen recognize as long-term capital gain income on his Form 1040, U.S. Individual Income Tax Return, for Year 3?

a.

$1,500

b.

$300

c.

$2,000

d.

$750

A

Choice “a” is correct. The receipt of a nontaxable stock dividend will require the shareholder to spread the basis of his original shares over both the original shares and the new shares received, resulting in the same total basis but a lower basis per share of stock held. Therefore, Allen’s total basis remains the same, $10,000, but is now split between 200 shares (a 2-for-1 split and he originally owned 100 shares). Therefore, his basis per share goes from $100/share ($10,000/100) to $50/share ($10,000/200). Consequently, his basis in the 100 shares sold is 100 x $50 = $5,000. Calculate his gain as follows:

Amount realized ($65 x 100) $ 6,500

Adjusted basis (5,000 - calculated above) (5,000)

Realized & recognized gain $ 1,500

Choices “b”, “d”, and “c” are incorrect.

How well did you know this?
1
Not at all
2
3
4
5
Perfectly
18
Q

Wallace purchased 500 shares of Kingpin, Inc. 15 years ago for $25,000. Wallace has worked as an owner/employee and owned 40% of the company throughout this time. This year, Kingpin, which is not an S corporation, redeemed 100% of Wallace’s stock for $200,000. What is the treatment and amount of income or gain that Wallace should report?

a.

$200,000 long-term capital gain.

b.

$175,000 long-term capital gain.

c.

$175,000 ordinary income.

d.

$0

A

Choice “b” is correct. An investment in a capital asset (e.g., stock) results in the income being capital (either a capital loss or a capital gain). Ownership percentage is not a factor in the calculation, and, in this question, nor is the fact that the corporation is not an S corporation. The calculation is simple: Wallace invested $25,000 in the stock and received $200,000 for 100% of his investment 15 years later. The capital gain is $175,000 ($200,000 - $25,000), and it is considered long-term because the stock was held for greater than one year.

Choice “d” is incorrect. There is $175,000 of gain on the transaction ($200,000 - $25,000). This type of transaction is not a transaction that is excluded from tax in the tax code.

Choice “c” is incorrect. An investment in a capital asset (e.g., stock) results in the income being capital (either a capital loss or a capital gain). Although the calculation of the income is correct (i.e., $175,000), ordinary income is not the proper treatment for this transaction.

Choice “a” is incorrect. Although this transaction does result in a long-term capital gain, Wallace has basis in the stock ($25,000), and the gain is calculated as the proceeds from the sale ($200,000) less the basis in the stock.

How well did you know this?
1
Not at all
2
3
4
5
Perfectly
19
Q

Which of the following sales should be reported as a capital gain?

a.

Sale of equipment.

b.

Sale of inventory.

c.

Government bonds sold by an individual investor.

d.

Real property subdivided and sold by a dealer.

A

Choice “c” is correct. Government bonds held by an individual investor are considered capital assets in the hands of the investor. When these types of security investments are sold, the resulting gain or loss is reported as capital.

Choice “a” is incorrect. In this case, we must assume that the BEST answer is option “c” (as that option would ALWAYS result in capital gain or loss treatment) and that the examiners are assuming that the equipment is depreciable equipment that has been used in a business for over one year. [If the equipment had been considered a personal asset by the examiners and had sold for a gain, it would also be a capital asset that sold for a capital gain, and there would be two correct answers. Remember that the correct answer is the option that best answers the question.] Depreciable equipment used in a business and held for over one year falls under the category of Section 1245 property. When Section 1245 assets are sold at a gain, all the accumulated depreciation on the asset is recaptured as ordinary income (the same category as the depreciation expense was deducted against), and any remaining gain (typically, in practice, this is not the case, though, as the asset would have had to sell for an amount greater than its purchase price) is capital gain under Code Section 1231. [Note that Section 1245 applies only to gains. If the asset had sold for a loss, the loss would have been ordinary under Section 1231.]

Choice “d” is incorrect. Real property sold by a dealer is considered inventory and results in ordinary income or ordinary losses upon sale. Inventory is not a capital asset and is not afforded the capital gain benefits.

Choice “b” is incorrect. Inventory is not a capital asset and is not afforded the capital gain benefits. The sale of inventory results in ordinary income or loss (e.g., gross profit on sales) being reported on the tax return, as inventory is an asset held for sale in the ordinary course of business.

How well did you know this?
1
Not at all
2
3
4
5
Perfectly
20
Q

Starr, a self-employed individual, purchased a piece of equipment for use in Starr’s business. The costs associated with the acquisition of the equipment were:

Purchase price $ 55,000

Delivery charges 725

Installation fees 300

Sales tax 3,400

What is the depreciable basis of the equipment?

a.

$59,125

b.

$58,400

c.

$59,425

d.

$55,000

A

Choice “c” is correct. The rules for depreciable basis in tax are generally the same as the GAAP rules for capitalizing an asset. The depreciable basis is the cost associated with the purchase of the asset and with getting the asset ready for its intended use. Further improvements are also capitalized, and the basis is reduced for any accumulated depreciation. In this case, the cost of obtaining the equipment and getting the equipment ready for its intended use includes all the items shown above, as follows:

Purchase price $ 55,000

Delivery charges 725

Installation fees 300

Sales tax 3,400

Total depreciable basis $ 59,425

Choice “d” is incorrect. The costs of delivery charges, installation, and sales tax are all part of the cost of obtaining the asset and getting the asset ready for its intended use. All of these charges are included in the depreciable basis of the equipment.

Choice “b” is incorrect. The costs of delivery charges and installation are both part of the cost of obtaining the asset and getting the asset ready for its intended use. These charges are included in the depreciable basis of the equipment.

Choice “a” is incorrect. The cost of installation is part of the cost getting the asset ready for its intended use. This charge is included in the depreciable basis of the equipment.

How well did you know this?
1
Not at all
2
3
4
5
Perfectly
21
Q

Which of the following statements is the best definition of real property?

a.

Real property is land and everything permanently attached to it.

b.

Real property is only land.

c.

Real property is land and intangible property in realized form.

d.

Real property is all tangible property including land.

A

Choice “a” is correct. Real property includes land and all items permanently affixed to the land (e.g., buildings, paving, etc.)

Choice “b” is incorrect. Real property includes more than just the land (as per the explanation above); it includes all items permanently affixed to land.

Choice “d” is incorrect. “All” tangible property could include moveable personal property and is therefore, incorrect.

Choice “c” is incorrect. “Intangible property in realized form” is a distracter and a contradiction in terms.

How well did you know this?
1
Not at all
2
3
4
5
Perfectly
22
Q

Gibson purchased stock with a fair market value of $14,000 from Gibson’s adult child for $12,000. The child’s cost basis in the stock at the date of sale was $16,000. Gibson sold the same stock to an unrelated party for $18,000. What is Gibson’s recognized gain from the sale?

a.

$2,000

b.

$6,000

c.

$4,000

d.

$0

A

Choice “a” is correct. Losses are disallowed on most related party sales transactions even if they were made at an arm’s length (FMV) price. The basis (and related gain or loss) of the (second) buying relative depends on whether the second relative’s resale price is higher, lower, or between the first relative’s basis and the lower selling price to the second relative. In this case, the $4,000 capital loss on the sale by Gibson’s adult child to Gibson [$12,000 SP - $16,000 Basis] is disallowed. Gibson’s basis is determined by his selling price to a third party. In this case, the selling price is $18,000, which is HIGHER than the original basis of Gibson’s adult child. Gibson’s basis in the stock is, therefore, his adult child’s basis of $16,000. Gibson’s recognized basis is calculated as follows:

Selling price $ 18,000

Basis (16,000)

Gain $ 2,000

Choice “d” is incorrect. There would be a zero gain or loss if the selling price were between the adult child’s basis and Gibson’s purchase price, but this is not the case in the facts.

Choice “c” is incorrect. This answer option uses the fair market value of the stock at the date of purchase as the basis. As is discussed above, the rules do not provide for this treatment. [$18,000 SP - $14,000 FMV = $4,000]

Choice “b” is incorrect. This would be the answer if the basis were Gibson’s purchase price of $12,000; however, because the stock sold for more than Gibson’s child’s basis and the child had a disallowed loss on the sale to Gibson, Gibson is allowed to use his child’s original basis of $16,000 as his basis for the stock on the date of the second sale. [$18,000 SP - $12,000 PP = $6,000]

How well did you know this?
1
Not at all
2
3
4
5
Perfectly
23
Q

An individual had the following capital gains and losses for the year:

Short-term capital loss $ 70,000

Long-term gain (unrecaptured Section 1250 at 25%) 56,000

Collectibles gain (28% rate) 10,000

Long-term gain (15% rate) 20,000

What will be the net gain (loss) reported by the individual and at what applicable tax rate(s)?

a.

Short-term loss of $3,000 at the ordinary rate and long-term capital gain of $86,000 at the 15% rate.

b.

Long-term gain of $16,000 at the 15% rate.

c.

Long-term capital gain of $3,000 at the 15% rate, collectibles gain of $10,000 at the 28% rate, and Section 1250 gain of $56,000 at the 25% rate.

d.

Short-term loss of $3,000 at the ordinary rate, long-term capital gain of $10,000 at the 15% rate, collectibles gain of $10,000 at the 28% rate, and Section 1250 gain of $56,000 at the 25% rate.

A

Choice “b” is correct. Specific netting procedures for capital gains and losses are outlined in the Internal Revenue Code for non-corporate taxpayers. Gains and losses are netted within each tax rate group (e.g., the 15% rate group). The facts of this question have already performed this step for us.

Short-term Capital Gains and Losses

If there are any short-term capital losses (this includes any short-term capital loss carryovers), they are first offset against any short-term gains that would be taxable at the ordinary income rates.

Any remaining short-term capital loss is used to offset any long-term capital gains from the 28% rate group (e.g., collectibles).

Any remaining short-term capital loss is then used to offset any long-term gains from the 25% group (e.g., un-recaptured Section 1250 gains).

Any remaining short-term capital loss is used to offset any long-term capital gains applicable at the lower (e.g., 15%) tax rate.

Long-term Capital Gains and Losses

If there are any long-term capital losses (this includes any long-term capital loss carryovers) from the 28% rate group, they are first offset against any net gains from the 25% rate group and then against net gains from the 15% rate group.

If there are any long-term capital losses (this includes any long-term capital loss carryovers) from the 15% rate group, they are offset first against any net gains from the 28% rate group and then against net gains from the 25% rate group.

In this case, we are given net short-term capital losses of $70,000 to start with. Following the rules above, this first goes to offset any short-term gains at the ordinary income rates, but there are none in the facts. So, the next step is to offset the losses against any 28% rate gain long-term capital gains. The facts provide that there is $10,000 in gains from collectibles (taxable at the 28% rate). The remaining short-term loss ($60,000) is next used to offset the long-term capital gains at the 25% rate. The facts give us un-recaptured Section 1250 gains of $56,000 (taxed at the 25% tax rate). The remaining short-term capital loss is $4,000 ($70,000 - $10,000 - $56,000 = $4,000). The balance of the short-term capital losses is finally used to offset any capital gains taxed at the 15% tax rate, which the facts give us as $20,000. Therefore, after the $4,000 remaining short-term capital loss is applied to offset the $20,000 long-term capital gain taxed at the 15% tax rate, there is an amount of $16,000 remaining of long-term capital gains to be taxed at the 15% tax rate.

Choices “a”, “c”, and “d” are incorrect, per the ordering rules discussed above.

How well did you know this?
1
Not at all
2
3
4
5
Perfectly
24
Q

Dunn received 100 shares of stock as a gift from Dunn’s grandparent. The stock cost Dunn’s grandparent $32,000 and it was worth $27,000 at the time of the transfer to Dunn. Dunn sold the stock for $29,000. What amount of gain or loss should Dunn report from the sale of the stock?

a.

$3,000 gain.

b.

$3,000 loss.

c.

$2,000 gain.

d.

$0

A

Choice “d” is correct. To determine the amount of gain or loss that should be reported on the sale of gifted property, a determination must be made as to whether the property is sold at a gain or a loss. The stock in this question has a $27,000 value which is less than its $32,000 cost. The basis for gain is the adjusted basis of the donor on the date of gift, or $32,000. However, the stock is sold for $29,000, which is not at a gain. The basis for loss is the lower of the adjusted basis or the fair market value on the date of gift, or $27,000. However, the stock is not sold at a loss. In this situation, neither gain nor loss is recognized, and the “middle” basis of the subsequent sales price is used.

Choice “c” is incorrect. The stock is not sold at a gain. The basis would have had to be $27,000 for the stock to be sold at a $2,000 gain ($29,000 - $27,000). That is the fair market value on the date of gift, but it is not the basis used when the property gifted is sold at a gain.

Choice “a” is incorrect. The stock is not sold at a gain. The basis would have had to be $26,000 for the stock to be sold at a $3,000 gain ($29,000 - $26,000). It is difficult to determine how that basis could have been calculated.

Choice “b” is incorrect. The stock is not sold at a loss. The basis would have had to be $32,000 for the stock to be sold at a $3,000 loss ($29,000 - $32,000). That is the adjusted basis of the stock, but it is not the basis used when the property gifted is sold at a loss.

How well did you know this?
1
Not at all
2
3
4
5
Perfectly
25
Q

Which of the following items is a capital asset?

a.

Real property used in a trade or business.

b.

Accounts receivable for inventory sold.

c.

Depreciable business property.

d.

An automobile for personal use.

A

Choice “d” is correct. An automobile for personal use is a capital asset.

Choice “c” is incorrect. Depreciable business property is a Section 1231 asset, not a capital asset.

Choice “b” is incorrect. Accounts receivable for inventory sold is an ordinary income asset, just like the original inventory.

Choice “a” is incorrect. Real property used in a trade or business is a Section 1231 asset, not a capital asset.

How well did you know this?
1
Not at all
2
3
4
5
Perfectly
26
Q

Which of the following is a capital asset?

a.

Inventory held primarily for sale to customers.

b.

Land held as an investment.

c.

Accounts receivable.

d.

A computer system used by the taxpayer in a personal accounting business.

A

Rule: Capital assets include property (real and personal) held by the taxpayer forinvestment, such as:

Personal automobile of the taxpayer

Furniture and fixtures in the home of the taxpayer

Stocks and securities of all types (except those held by dealers)

Personal property of a taxpayer not used in a trade or business

Real property not used in a trade or business

Interest in a partnership

Goodwill of a corporation

Copyrights, literary, musical, or artistic compositions purchased

Other assets held for investment

Items that are NOT capital assets include:

Property normally included in inventory or held for sale to customers in the ordinary course of business

Depreciable personal property and real estate used in a trade or business

Accounts and notes receivable arising from sales or services in the taxpayer’s business

Copyrights, literary, musical, or artistic compositions held by the originalartist (with the exception of musical compositions held by the original artist)

Treasury stock (not an ordinary asset and not subject to capital gains treatment)

Choice “b” is correct. Per the above information and rule, real property not used in a trade or business (e.g., land held for investment) is a capital asset.

Choice “a” is incorrect. Per the above information, property normally included in inventory or held for sale to customers in the ordinary course of business is NOT a capital asset.

Choice “c” is incorrect. Per the above information, accounts and notes receivable arising from sales or services in the taxpayer’s business are NOT capital assets.

Choice “d” is incorrect. Per the above information, depreciable personal property and real estate used in a trade or business (such as a computer system) is NOT a capital asset.

How well did you know this?
1
Not at all
2
3
4
5
Perfectly
27
Q

A heavy equipment dealer would like to trade some business assets in a nontaxable exchange. Which of the following exchanges would qualify as nontaxable?

a.

A road grader held in inventory for another road grader.

b.

The company jet for a large truck to be used in the corporation.

c.

Investment securities for antiques to be held as investments.

d.

A corporate office building for a vacant lot.

A

Rule: Nonrecognition treatment is accorded to a “like-kind” exchange of property used in the trade or business or held for investment (with the exception of inventory, stock, securities, partnership interests, and real property in different countries). “Like-kind” means the same type of investment (e.g., realty for realty or personalty for personalty, assuming the personal property falls within the same “asset class” for tax depreciation purposes).

Choice “d” is correct. The exchange of a corporate office building for a vacant lot qualifies for like-kind nonrecognition treatment. It is the exchange of realty for realty of property used in the trade or business or held for investment.

Choice “b” is incorrect. Although this answer option exchanges personal property used in a trade or business, the “asset classes” for the corporate jet and the heavy equipment differ.

Choice “c” is incorrect. The exchange of investment securities for antiques to be held as investments does not qualify for nonrecognition treatment. It is one of the exceptions identified in the rule, above.

Choice “a” is incorrect. The exchange of a road grader held in inventory for another road grader does not qualify for nonrecognition treatment. It is one of the exceptions identified in the rule, above.

How well did you know this?
1
Not at all
2
3
4
5
Perfectly
28
Q

Bluff purchased equipment for business use for $35,000 and made $1,000 of improvements to the equipment. After deducting depreciation of $5,000, Bluff gave the equipment to Russett for business use. At the time the gift was made, the equipment had a fair market value of $32,000. Ignoring gift tax consequences, what is Russett’s basis in the equipment?

a.

$36,000

b.

$32,000

c.

$31,000

d.

$35,000

A

Rule: Property acquired as a gift generally retains the rollover cost basis as it had in the hands of the donor at the time of the gift. Basis is increased by any gift tax paid attributable to the appreciation in the value of the gift (but the facts in this case indicate to ignore gift tax consequences). There is an exception to the general rule: if the fair market value at the date of gift is lower than the roll-over cost basis from the donor, the basis for the donee depends upon the donee’s future selling price of the asset.

The asset may sell for (1) greater than the donor’s basis, (2) between the donor’s basis and the lower FMV at the date of gift, or (3) less than the FMV at the date of gift.

Choice “c” is correct. The first step is to determine the donor’s basis in the asset at the gift date. In this case, the basis is $31,000 ($35,000 + $1,000 - $5,000). The fair market value of the asset is $32,000 at the date of gift, which is greater than the donor’s basis, so the general rule applies. Property acquired as a gift generally retains the rollover cost basis as it had in the hands of the donor at the time of the gift. Thus, Russett’s basis in the equipment is $31,000.

Choice “b” is incorrect. The first step is to determine the donor’s basis in the asset at the gift date. In this case, the basis is $31,000 ($35,000 + $1,000 - $5,000). The fair market value of the asset is $32,000 at the date of gift, which is greater than the donor’s basis, so the general rule applies. Property acquired as a gift generally retains the rollover cost basis as it had in the hands of the donor at the time of the gift. Thus, Russett’s basis in the equipment is $31,000 (the cost basis of the donor), not the $32,000 fair market value at the date of gift.

Choice “d” is incorrect. This answer option incorrectly assumes the basis is only the $35,000 purchase price of the asset and ignores the $1,000 in improvements and the basis reduction for the $5,000 in accumulated depreciation.

Choice “a” is incorrect. This answer option incorrectly assumes the basis is only the $35,000 purchase price of the asset plus the $1,000 in improvements and ignores the basis reduction for the $5,000 in accumulated depreciation.

How well did you know this?
1
Not at all
2
3
4
5
Perfectly
29
Q

A taxpayer trades in an automobile used solely for business purposes for another automobile to be used in his business. The automobile originally cost $35,000 and he has taken $18,000 in depreciation. The old automobile is currently worth $20,000 and the new automobile the taxpayer wants in exchange is worth $20,000. No other cash or property is exchanged in the transaction. What is the gain or loss realized by the taxpayer on this transaction?

a.

$0

b.

$3,000 loss

c.

$3,000 gain

d.

$15,000 loss

A

Calculations for “Realized Gain with No Boot”

Gain/Loss Realized:

Amount realized

=

Fair market value of auto received − Adjusted basis of auto given up

=

$20,000 fair market value of new auto − ($35,000 cost − $18,000 depreciation)

=

$20,000 fair market value of new auto − $17,000 adjusted basis of old auto

=

$3,000 gain

Gain/Loss Recognized:

Gain recognized

=

$0 (the lesser of gain realized of $3,000 or boot received of $0)

Basis of New Property:

New basis

=

Adjusted basis of property given up + Gain recognized

=

$17,000 + $0

=

$17,000

Alternate calculation: $20,000 FMV new property − $3,000 deferred gain = $17,000 basis new property.

Choice “c” is correct. A $3,000 gain is realized on the transaction [fair market value of the new auto, $20,000 − $17,000, the adjusted basis of the old auto ($35,000 cost − $18,000 accumulated depreciation)].

Choice “a” is incorrect. $0 is the gain/loss recognized (the lesser of gain realized of $3,000 or boot received of $0), not the gain realized. This is also the difference in fair market value between old and new autos.

Choice “b” is incorrect. There is a $3,000 gain, not a loss [fair market value of the new auto, $20,000 − $17,000, the adjusted basis of the old auto ($35,000 cost − $18,000 accumulated depreciation)].

Choice “d” is incorrect. $15,000 loss is the difference between the $35,000 cost of the old auto and $20,000 fair market value of the new auto. This calculation ignores the accumulated depreciation in determining the carrying value of the old auto.

How well did you know this?
1
Not at all
2
3
4
5
Perfectly
30
Q

A taxpayer is trading in an automobile used solely for business purposes for another automobile to be used in his business. The automobile originally cost $35,000 and he has taken $18,000 in depreciation. The old automobile is currently worth $20,000 and the new automobile the taxpayer wants in exchange is worth $20,000. No other cash or property is exchanged in the transaction. What is the gain or loss recognized by the taxpayer on this transaction?

a.

$3,000 gain

b.

$0

c.

$15,000 loss

d.

$3,000 loss

A

Gain/Loss Realized:

Amount realized

=

Fair market value of auto received − Adjusted basis of auto given up

=

$20,000 fair market value of new auto − ($35,000 cost − $18,000 depreciation)

=

$20,000 fair market value of new auto − $17,000 adjusted basis of old auto

=

$3,000 gain

Gain/Loss Recognized:

Gain recognized

=

$0 (the lesser of gain realized of $3,000 or boot received of $0)

Basis of New Property:

=

New basis

=

Adjusted basis of property given up + Gain recognized

=

$17,000 + $0

=

$17,000

Alternate calculation: $20,000 FMV new property − $3,000 deferred gain = $17,000 basis of new property.

Choice “b” is correct. $0 is the recognized (lesser of gain realized of $3,000 or boot received of $0).

Choice “d” is incorrect. There is a $3,000 gain, not a loss [fair market value of the new auto, $20,000 − $17,000, the adjusted basis of the old auto ($35,000 cost − $18,000 accumulated depreciation)].

Choice “a” is incorrect. A $3,000 gain is realized on the transaction, but is not recognized [fair market value of the new auto, $20,000 − $17,000, the adjusted basis of the old auto ($35,000 cost − $18,000 accumulated depreciation)].

Choice “c” is incorrect. $15,000 loss is the difference between the $35,000 cost of the old auto and $20,000 fair market value of the new auto. This calculation ignores the accumulated depreciation in determining the carrying value of the old auto.

How well did you know this?
1
Not at all
2
3
4
5
Perfectly
31
Q

A taxpayer is trading in an automobile used solely for business purposes for another automobile to be used in his business. The automobile originally cost $35,000 and he has taken $18,000 in depreciation. The old automobile is currently worth $20,000 and the new automobile the taxpayer wants in exchange is worth $20,000. No other cash or property is exchanged in the transaction. What is the taxpayer’s basis in the new automobile received?

a.

$17,000

b.

$20,000

c.

$18,000

d.

$35,000

A

Calculations for “New Basis on Like Kind-Exchange Property with No Boot”

Gain/Loss Realized:

Amount realized

=

Fair market value of auto received − Adjusted basis of auto given up

=

$20,000 fair market value of new auto − ($35,000 cost − $18,000 depreciation)

=

20,000 fair market value of new auto − $17,000 adjusted basis of old auto

=

$3,000 gain

Gain/Loss Recognized:

Gain recognized

=

$0 (the lesser of gain realized of $3,000 or boot received of $0)

Basis of New Property:

New basis

=

Adjusted basis of property given up + Gain recognized

=

$17,000 + $0

=

$17,000

Alternate calculation: $20,000 FMV new property − $3,000 deferred gain = $17,000 basis of new property.

Choice “a” is correct. $17,000 is the substituted basis [the adjusted basis of the old auto ($35,000 cost − $18,000 accumulated depreciation) + gain recognized $0].

Choice “c” is incorrect. $18,000 is accumulated depreciation on the old auto.

Choice “b” is incorrect. $20,000 is the fair market value of both the old and new autos.

Choice “d” is incorrect. $35,000 is the original cost of the old auto.

How well did you know this?
1
Not at all
2
3
4
5
Perfectly
32
Q

A taxpayer is trading in an automobile used solely for business purposes for another automobile to be used in his business. The automobile originally cost $35,000 and he has taken $12,000 in depreciation. The old automobile is currently worth $20,000 and the new automobile the taxpayer wants in exchange is worth $20,000. No other cash or property is exchanged in the transaction. What is the gain or loss realized by the taxpayer on this transaction?

a.

$3,000 gain

b.

$3,000 loss

c.

$15,000 loss

d.

$0

A

Calculations for “Realized Loss with No Boot”

Gain/Loss Realized:

Amount realized

=

Fair market value of auto received − Adjusted basis of auto given up

=

$20,000 fair market value of new auto − ($35,000 cost − $12,000 depreciation)

=

$20,000 fair market value of new auto − $23,000 adjusted basis of old auto

=

$3,000 loss

Gain/Loss Recognized:

Loss recognized

=

$0 (Realized loss is never recognized in like-kind exchanges.)

Basis of New Property:

=

New basis

=

Adjusted basis of property given up

=

$23,000 + $0

=

$23,000

Alternate calculation: $20,000 FMV new property + $3,000 deferred loss = $23,000 basis of new property.

Choice “b” is correct. The taxpayer realizes a $3,000 loss (fair market value of new auto $20,000 − $23,000 adjusted basis of old auto).

Choice “d” is incorrect. $0 is the gain/loss recognized (the lesser of gain realized of $3,000 or boot received of $0), not the gain realized. This is also the difference in fair market value between old and new autos.

Choice “a” is incorrect. There is a $3,000 loss, not a $3,000 gain.

Choice “c” is incorrect. $15,000 loss is the difference between the $35,000 cost of the old auto and $20,000 fair market value of the new auto. This calculation ignores the accumulated depreciation in determining the carrying value of the old auto.

How well did you know this?
1
Not at all
2
3
4
5
Perfectly
33
Q

A taxpayer is trading in an automobile used solely for business purposes for another automobile to be used in his business. The automobile originally cost $35,000 and he has taken $12,000 in depreciation. The old automobile is currently worth $20,000 and the new automobile the taxpayer wants in exchange is worth $20,000. No other cash or property is exchanged in the transaction. What is the gain or loss recognized by the taxpayer on this transaction?

a.

$0

b.

$15,000 loss.

c.

$3,000 loss.

d.

$3,000 gain.

A

Calculations for “Recognized Loss with No Boot”

Gain/Loss Realized:

Amount realized

=

Fair market value of auto received − Adjusted basis of auto given up

=

$20,000 fair market value of new auto − ($35,000 cost − $12,000 depreciation)

=

$20,000 fair market value of new auto − $23,000 adjusted basis of old auto

=

$3,000 loss

Gain/Loss Recognized:

Loss recognized

=

$0 (Realized loss is never recognized in like-kind exchanges.)

Basis of New Property:

=

New basis

=

Adjusted basis of property given up

=

$23,000 + $0

=

$23,000

Alternate calculation: $20,000 FMV new proerty + $3,000 deferred loss = $23,000 basis of new property.

Choice “a” is correct. $0 is recognized (a realized loss is never recognized in like-kind exchange.)

Choice “c” is incorrect. A $3,000 loss is realized on the transaction, but is not recognized [fair market value of the new auto, $20,000 − $23,000, the adjusted basis of the old auto ($35,000 cost − $12,000 accumulated depreciation)].

Choice “d” is incorrect. There is a $3,000 realized loss, not a $3,000 gain. The $3,000 loss is also not recognized.

Choice “b” is incorrect. $15,000 loss is the difference between the $35,000 cost of the old auto and $20,000 fair market value of the new auto. This calculation ignores the accumulated depreciation in determining the carrying value of the old auto.

How well did you know this?
1
Not at all
2
3
4
5
Perfectly
34
Q

A taxpayer is trading in an automobile used solely for business purposes for another automobile to be used in his business. The automobile originally cost $35,000 and he has taken $12,000 in depreciation. The old automobile is currently worth $20,000 and the new automobile the taxpayer wants in exchange is worth $20,000. No other cash or property is exchanged in the transaction. What is the taxpayer’s basis in the new automobile received?

a.

$23,000

b.

$35,000

c.

$20,000

d.

$12,000

A

Gain/Loss Realized:

Amount realized

=

Fair market value of auto received − Adjusted basis of auto given up

=

$20,000 fair market value of new auto − ($35,000 cost − $12,000 depreciation)

=

$20,000 fair market value of new auto − $23,000 adjusted basis of old auto

=

$3,000 loss

Gain/Loss Recognized:

Loss recognized

=

$0 (Realized loss is never recognized in like-kind exchanges.)

Basis of New Property:

=

New basis

=

Adjusted basis of property given up

=

$23,000 + $0

=

$23,000

Alternate calculation: $20,000 FMV new property + $3,000 deferred loss = $23,000 basis of new property.

Choice “a” is correct. The basis for the new auto is $23,000, the adjusted basis of the old auto ($35,000 cost − $12,000 accumulated depreciation).

Choice “d” is incorrect. $12,000 is amount of accumulated depreciation on the old auto.

Choice “c” is incorrect. $20,000 is fair market value of both the old auto and the new auto.

Choice “b” is incorrect. $35,000 is the original cost of the old auto.

How well did you know this?
1
Not at all
2
3
4
5
Perfectly
35
Q

A taxpayer is trading in an automobile used solely for business purposes for another automobile to be used in his business. The automobile originally cost $35,000 and he has taken $18,000 in depreciation. The old automobile is currently worth $20,000 and the new automobile the taxpayer wants in exchange is worth $22,000, so the taxpayer has agreed to pay $2,000 cash in addition to the trade-in. What is the gain or loss realized by the taxpayer on this transaction?

a.

$0

b.

$2,000 gain

c.

$5,000 gain

d.

$3,000 gain

A

Gain/Loss Realized:

Amount realized

=

(Fair market value of auto received − boot paid) − Adjusted basis of auto given up

=

($22,000 fair market value new auto − $2,000 boot paid) − ($35,000 cost of old auto − $18,000 accumulated depreciation)

=

$20,000 − $17,000

=

$3,000 gain

Gain/Loss Recognized:

Gain recognized

=

$0 (lesser of realized gain of $3,000 or boot received of $0)

Basis of New Property:

=

New basis

=

Adjusted basis of property given up + Boot paid

=

$17,000 + $2,000

=

$19,000

Alternate calculation: $22,000 FMV new property − $3,000 deferred gain = $19,000 basis new property.

Choice “d” is correct. A $3,000 gain is realized on the transaction [fair market value of the new auto, $22,000 − boot paid of $2,000 − $17,000, the adjusted basis of the old auto ($35,000 cost − $18,000 accumulated depreciation)].

Choice “a” is incorrect. $0 is the gain/loss recognized (the lesser of the $3,000 gain realized or the boot received of $0), not the gain realized.

Choice “b” is incorrect. $2,000 is the difference between the fair market value of the old auto and the fair market value of the new auto, not the gain realized. It is also the amount of boot paid (not received).

Choice “c” is incorrect. $5,000 gain would be the gain realized, if the boot of $2,000 was ignored.

How well did you know this?
1
Not at all
2
3
4
5
Perfectly
36
Q

A taxpayer is trading in an automobile used solely for business purposes for another automobile to be used in his business. The automobile originally cost $35,000 and he has taken $18,000 in depreciation. The old automobile is currently worth $20,000 and the new automobile the taxpayer wants in exchange is worth $22,000, so the taxpayer has agreed to pay $2,000 cash in addition to the trade-in. What is the gain or loss recognized by the taxpayer on this transaction?

a.

$3,000 gain

b.

$0

c.

$2,000 gain

d.

$5,000 gain

A

Calculations for “Recognized Gain with Boot Paid in Cash or Non-Like-Kind Property”

Gain/Loss Realized:

Amount realized

=

(Fair market value of auto received - Boot paid) - Adjusted basis of auto given up

=

($22,000 fair market value new auto - $2,000 boot paid) -
($35,000 cost of old auto - $18,000 accumulated depreciation)

=

$20,000 - $17,000

=

$3,000 gain

Gain/Loss Recognized:

Gain recognized

=

$0 (lesser of realized gain of $3,000 or boot received of $0)

Basis of New Property:

=

New basis

=

Adjusted basis of property given up + boot paid

=

$17,000 + $2,000

=

$19,000

Alternate calculation: $22,000 FMV new property - $3,000 deferred gain = $19,000 basis of new property.

Choice “b” is correct. $0 of gain is recognized (the lesser of gain realized of $3,000 and boot received of $0).

Choice “c” is incorrect. $2,000 is the difference between the fair market value of the old auto and the fair market value of the new auto. It is also the amount of boot paid (not received).

Choice “a” is incorrect. $3,000 is the gain realized, not the gain recognized.

Choice “d” is incorrect. A $5,000 gain would be realized if the boot paid of $2,000 was ignored [fair market value of the new auto of $22,000 - adjusted basis of the old auto of $17,000 ($35,000 cost - $18,000 depreciation)].

How well did you know this?
1
Not at all
2
3
4
5
Perfectly
37
Q

A taxpayer is trading in an automobile used solely for business purposes for another automobile to be used in his business. The automobile originally cost $35,000 and he has taken $18,000 in depreciation. The old automobile is currently worth $20,000 and the new automobile the taxpayer wants in exchange is worth $22,000. The taxpayer has agreed to pay $2,000 cash in addition to the trade-in. What is the taxpayer’s basis in the new automobile received?

a.

$17,000

b.

$20,000

c.

$22,000

d.

$19,000

A

Calculations for “New Basis of Like-Kind Property with Liabilities Assumed (Boot Paid)”

Gain/Loss Realized:

Amount realized

=

(Fair market value of auto received - Boot paid) - Adjusted basis of auto given up

=

($22,000 fair market value new auto - $2,000 boot paid) - ($35,000 cost of old auto - $18,000 accumulated depreciation)

=

$20,000 - $17,000

=

$3,000 gain

Gain/Loss Recognized:

Gain recognized

=

$0 (lesser of realized gain of $3,000 or boot received of $0)

Basis of New Property:

=

New basis

=

Adjusted basis of property given up + boot paid

=

$17,000 + $2,000

=

$19,000

Alternate calculation: $22,000 FMV new property - $3,000 deferred gain = $19,000 basis of new property.

Choice “d” is correct. $19,000 is the basis of the new auto ($17,000 adjusted basis of the old auto ($35,000 cost - $18,000 accumulated depreciation) + $2,000 boot paid).

Choice “a” is incorrect. $17,000 is the adjusted basis of the old auto ($35,000 cost - $18,000 accumulated depreciation). The boot paid must also be included in the new basis.

Choice “b” is incorrect. $20,000 is the fair market value of the old auto.

Choice “c” is incorrect. $22,000 is the fair market value of the new auto.

How well did you know this?
1
Not at all
2
3
4
5
Perfectly
38
Q

A taxpayer is trading in an automobile used solely for business purposes for another automobile to be used in his business. The automobile originally cost $35,000 and he has taken $18,000 in depreciation. The old automobile is currently worth $20,000 and the new automobile the taxpayer wants in exchange is only worth $16,500. The other party agrees to give the taxpayer a trailer worth $3,500 in addition to the new auto. What is the gain or loss realized by the taxpayer on this transaction?

a.

$500 loss

b.

$3,000 gain

c.

$3,500 gain

d.

$3,500 loss

A

Calculations for “Realized Gain with Boot Received (Of Non-Like-Kind Property)”

Gain/Loss Realized:

Amount realized

=

Fair market value of new auto + Boot received - Adjusted basis of auto given up

=

$16,500 fair market value new auto + $3,500 fair market value of trailer - $17,000
adjusted basis of the old auto ($35,000 cost - $18,000 accumulated depreciation)

=

$3,000 gain

Gain/Loss Recognized:

Gain recognized

=

$3,000 (the lesser of realized gain of $3,000 or boot received of $3,500)

Basis of New Property:

New basis

=

Adjusted basis of property given up + Gain recognized - Boot received

=

$17,000 + $3,000 + $0 - $3,500

=

$16,500

Alternate calculation: $16,500 FMV new property + $0 deferred loss - $0 deferred gain = $16,500 basis of new property.

Choice “b” is correct. A $3,000 gain is realized on the transaction [fair market value of the new auto, $16,500 + $3,500 of boot received - $17,000, the adjusted basis of the old auto ($35,000 cost - $18,000 accumulated depreciation)].

Choice “c” is incorrect. $3,500 is fair market value of boot received, not the gain realized.

Choice “a” is incorrect. $500 loss would be realized if the boot received of $3,500 was ignored.

Choice “d” is incorrect. $3,500 is the difference between the fair market value of the old auto and the fair market value of the new auto. $3,500 is also the fair market value of the boot received.

How well did you know this?
1
Not at all
2
3
4
5
Perfectly
39
Q

A taxpayer is trading in an automobile used solely for business purposes for another automobile to be used in his business. The automobile originally cost $35,000 and he has taken $18,000 in depreciation. The old automobile is currently worth $20,000 and the new automobile the taxpayer wants in exchange is only worth $16,500. The other party agrees to give the taxpayer a trailer worth $3,500 in addition to the new auto. What is the gain or loss recognized by the taxpayer on this transaction?

a.

$3,500 gain.

b.

$3,500 loss.

c.

$0

d.

$3,000 gain.

A

Calculations for “Recognized Gain with Boot Received (Of Non-Like-Kind Property)”

Gain/Loss Realized:

Amount realized

=

Fair market value of new auto + Boot received - Adjusted basis of auto given up

=

$16,500 fair market value new auto + $3,500 fair market value of trailer - $17,000
adjusted basis of the old auto ($35,000 cost - $18,000 accumulated depreciation)

=

$3,000 gain

Gain/Loss Recognized:

Gain recognized

=

$3,000 (the lesser of realized gain of $3,000 or boot received of $3,500)

Basis of New Property:

New basis

=

Adjusted basis of property given up + Gain recognized - Boot received

=

$17,000 + $3,000 + $0 - $3,500

=

$16,500

Alternate calculation: $16,500 FMV new property + 0 deferred loss - $0 deferred gain = $16,500 basis of new property.

Choice “d” is correct. A gain of $3,000 is recognized on the transaction (the lesser of $3,000 gain realized and $3,500 fair market value of boot received).

Choice “a” is incorrect. $3,500 is the fair market value of boot received. The gain recognized is lesser of gain realized and fair market value boot received.

Choice “c” is incorrect. $0 would be the gain recognized if boot was paid instead of received.

Choice “b” is incorrect. A $3,500 loss would result from a monetary exchange of assets with values of $20,000 and $16,500 for the old and new assets, respectively.

How well did you know this?
1
Not at all
2
3
4
5
Perfectly
40
Q

A taxpayer is trading in an automobile used solely for business purposes for another automobile to be used in his business. The automobile originally cost $35,000 and he has taken $18,000 in depreciation. The old automobile is currently worth $20,000 and the new automobile the taxpayer wants in exchange is only worth $16,500. The other party agrees to give the taxpayer a trailer worth $3,500 in addition to the new auto. What is the taxpayer’s basis in the new automobile received?

a.

$20,000

b.

$23,500

c.

$16,500

d.

$17,000

A

Calculations for “New Basis of Like-Kind Property with Liabilities Assumed (Boot Paid)”

Gain/Loss Realized:

Amount realized

=

Fair market value of new auto + Boot received - Adjusted basis of auto given up

=

$16,500 fair market value new auto + $3,500 fair market value of trailer - $17,000
adjusted basis of the old auto ($35,000 cost - $18,000 accumulated depreciation)

=

$3,000 gain

Gain/Loss Recognized:

Gain recognized

=

$3,000 (the lesser of realized gain of $3,000 or boot received of $3,500)

Basis of New Property:

New basis

=

Adjusted basis of property given up + Gain recognized - Boot received

=

$17,000 + $3,000 + $0 - $3,500

=

$16,500

Alternate calculation: $16,500 FMV new property + $0 deferred loss - $0 deferred gain = $16,500 basis of new property.

Choice “c” is correct. The basis of the new auto is $16,500 ($17,000 adjusted basis of the old auto ($35,000 cost - $18,000 accumulated depreciation) + $3,000 gain recognized - $3,500 boot received). It is also the fair market value of new auto.

Choice “d” is incorrect. $17,000 is the adjusted basis of the old auto ($35,000 cost - $18,000 accumulated depreciation), which ignores the boot received and the gain recognized.

Choice “a” is incorrect. $20,000 is the fair market value of the old auto.

Choice “b” is incorrect. $23,500 is the $3,500 boot received + $17,000 adjusted basis of the old auto + $3,000 gain recognized. The boot received is subtracted, not added.

41
Q

A taxpayer is trading in an automobile used solely for business purposes for another automobile to be used in his business. The automobile originally cost $35,000 and he has taken $18,000 in depreciation. The old automobile is currently worth $20,000 and the new automobile the taxpayer wants in exchange is only worth $17,500. The other party agrees to give the taxpayer $2,500 in cash in addition to the new auto. What is the gain or loss realized by the taxpayer on this transaction?

a.

$3,000 gain

b.

$2,500 gain

c.

$500 gain

d.

$2,500 loss

A

Calculations for “Realized Gain with Boot Received (Of Cash)”

Gain/Loss Realized:

Amount realized

=

Fair market value of new auto + Boot received - Adjusted basis of auto given up

=

$17,500 fair market value new auto + $2,500 cash boot - $17,000 adjusted
basis of the old auto ($35,000 cost - $18,000 accumulated depreciation)

=

$3,000 gain

Gain/Loss Recognized:

Gain recognized

=

$2,500 (the lesser of realized gain of $3,000 or boot received of $2,500)

Basis of New Property:

New basis

=

Adjusted basis of property given up + Gain recognized - Boot received

=

$17,000 + $2,500 + $0 - $2,500

=

$17,000

Alternate calculation: $17,500 FMV new property - $500 deferred gain = $17,000 basis of new property.

Choice “a” is correct. A $3,000 gain is realized on the transaction [fair market value of the new auto, $17,500 + $2,500 of boot received - $17,000, the adjusted basis of the old auto ($35,000 cost - $18,000 accumulated depreciation)].

Choice “b” is incorrect. $2,500 is the gain recognized, not the gain realized.

Choice “c” is incorrect. $500 would be the gain realized if the boot of $2,500 was ignored.

Choice “d” is incorrect. A $2,500 loss would result from a monetary exchange of assets with values of $20,000 and $17,500 for the old and new assets, respectively.

42
Q

A taxpayer is trading in an automobile used solely for business purposes for another automobile to be used in his business. The automobile originally cost $35,000 and he has taken $18,000 in depreciation. The old automobile is currently worth $20,000 and the new automobile the taxpayer wants in exchange is only worth $17,500. The other party agrees to give the taxpayer $2,500 in cash in addition to the new auto. What is the gain or loss recognized by the taxpayer on this transaction?

a.

$0

b.

$3,000 gain

c.

$2,500 loss

d.

$2,500 gain

A

Calculations for “Recognized Gain with Boot Received (Of Cash)”

Gain/Loss Realized:

Amount realized

=

Fair market value of new auto + Boot received - Adjusted basis of auto given up

=

$17,500 fair market value new auto + $2,500 cash boot - $17,000 adjusted
basis of the old auto ($35,000 cost - $18,000 accumulated depreciation)

=

$3,000 gain

Gain/Loss Recognized:

Gain recognized

=

$2,500 (the lesser of realized gain of $3,000 or boot received of $2,500)

Basis of New Property:

New basis

=

Adjusted basis of property given up + Gain recognized - Boot received

=

$17,000 + $2,500 + $0 - $2,500

=

$17,000

Alternate calculation: $17,500 FMV new property - $500 deferred gain = $17,000 basis of new property.

Choice “d” is correct. $2,500 gain recognized (the lesser of $3,000 gain realized and $2,500 cash boot received).

Choice “b” is incorrect. $3,000 is the amount of gain realized. The gain recognized is the lesser of the gain realized and the boot received.

Choice “a” is incorrect. $0 would be the gain recognized if the boot was paid instead of received,

Choice “c” is incorrect. A $2,500 loss would result from a monetary exchange of assets with values of $20,000 and $17,500 for the old and new assets, respectively.

43
Q

A taxpayer is trading in an automobile used solely for business purposes for another automobile to be used in his business. The automobile originally cost $35,000 and he has taken $18,000 in depreciation. The old automobile is currently worth $20,000 and the new automobile the taxpayer wants in exchange is only worth $17,500. The other party agrees to give the taxpayer $2,500 in cash in addition to the new auto. What is the taxpayer’s basis in the new automobile received?

a.

$17,000

b.

$20,000

c.

$17,500

d.

$22,000

A

Calculations for “New Basis of Like-Kind Property with Liabilities Assumed (Boot Paid)”

Gain/Loss Realized:

Amount realized

=

Fair market value of new auto + Boot received - Adjusted basis of auto given up

=

$17,500 fair market value new auto + $2,500 cash boot − $17,000 adjusted
basis of the old auto ($35,000 cost − $18,000 accumulated depreciation)

=

$3,000 gain

Gain/Loss Recognized:

Gain recognized

=

$2,500 (the lesser of realized gain of $3,000 or boot received of $2,500)

Basis of New Property:

New basis

=

Adjusted basis of property given up + Gain recognized − Boot received

=

$17,000 + $2,500 + $0 − $2,500

=

$17,000

Alternate calculation: $17,500 FMV new property - $500 deferred gain = $17,000 basis of new property.

Choice “a” is correct. The basis of the new auto is $17,000 ($17,000 adjusted basis of the old auto ($35,000 cost − $18,000 accumulated depreciation) + $2,500 gain recognized − $2,500 boot received).

Choice “c” is incorrect. $17,500 is fair market value of the new auto.

Choice “b” is incorrect. $20,000 is the fair market of the old auto.

Choice “d” is incorrect. $22,000 would be the new basis if the $2,500 boot was added, instead of subtracted.

44
Q

A taxpayer is trading in an automobile used solely for business purposes for another automobile to be used in his business. The automobile originally cost $35,000 and he has taken $12,000 in depreciation. The old automobile is currently worth $20,000 and the new automobile the taxpayer wants in exchange is only worth $16,500. The other party agrees to give the taxpayer $3,500 in cash in addition to the new auto. What is the gain or loss realized by the taxpayer on this transaction?

a.

$3,500 loss

b.

$3,000 loss

c.

$6,500 loss

d.

$3,500 gain

A

Calculations for “Realized Loss with Boot Received (Of Cash)”

Gain/Loss Realized:

Amount realized

=

Fair market value of new auto + Boot received - Adjusted basis of auto given up

=

$16,500 fair market value new auto + $3,500 cash boot - $23,000 adjusted basis
of the old auto ($35,000 cost - $12,000 accumulated depreciation)

=

$3,000 loss

Gain/Loss Recognized:

Loss recognized

=

$0 (Realized loss is never recognized in like-kind exchanges.)

Basis of New Property:

New basis

=

Adjusted basis of property given up - Loss recognized - Boot received

=

$23,000 - $0 - $3,500

=

$19,500

Alternate calculation: $16,500 FMV new property + $3,000 deferred loss = $19,500 basis of new property.

Choice “b” is correct. The taxpayer realizes a $3,000 loss (fair market value of new auto $16,500 + $3,500 boot received - $23,000 adjusted basis of old auto).

Choices “d” and “a” are incorrect. $3,500 is the amount of cash boot received.

Choice “c” is incorrect. A $6,500 loss would be realized if the boot of $3,500 were ignored [fair market value of the new auto of $16,500 - adjusted basis of the old auto of $23,000 ($35,000 cost - $12,000 accumulated depreciation)].

45
Q

A taxpayer is trading in an automobile used solely for business purposes for another automobile to be used in his business. The automobile originally cost $35,000 and he has taken $12,000 in depreciation. The old automobile is currently worth $20,000 and the new automobile the taxpayer wants in exchange is only worth $16,500. The other party agrees to give the taxpayer $3,500 in cash in addition to the new auto. What is the gain or loss recognized by the taxpayer on this transaction?

a.

$0

b.

$3,500 loss

c.

$3,000 loss

d.

$3,500 gain

A

Calculations for “Recognized Loss with Boot Received (Of Cash)”

Gain/Loss Realized:

Amount realized

=

Fair market value of new auto + Boot received - Adjusted basis of auto given up

=

$16,500 fair market value new auto + $3,500 cash boot - $23,000 adjusted basis
of the old auto ($35,000 cost - $12,000 accumulated depreciation)

=

$3,000 loss

Gain/Loss Recognized:

Loss recognized

=

$0 (Realized loss is never recognized in like-kind exchanges.)

Basis of New Property:

New basis

=

Adjusted basis of property given up - Loss recognized - Boot received

=

$23,000 - $0 - $3,500

=

$19,500

Alternate calculation: $16,500 FMV new property + $3,000 deferred loss = $19,500 basis of new property.

Choice “a” is correct. $0 is recognized (Losses are never recognized in like-kind exchanges.)

Choice “c” is incorrect. A $3,000 loss is realized on the transaction, but is not recognized [fair market value of the new auto, $20,000 - $17,000, the adjusted basis of the old auto ($35,000 cost - $18,000 accumulated depreciation)].

Choices “d” and “b” are incorrect. $3,500 is amount of cash boot received.

46
Q

A taxpayer is trading in an automobile used solely for business purposes for another automobile to be used in his business. The automobile originally cost $35,000 and he has taken $12,000 in depreciation. The old automobile is currently worth $20,000 and the new automobile the taxpayer wants in exchange is only worth $16,500. The other party agrees to give the taxpayer $3,500 in cash in addition to the new auto. What is the taxpayer’s basis in the new automobile received?

a.

$23,000

b.

$26,500

c.

$19,500

d.

$16,500

A

Calculations for “New Basis on Like-Kind Exchange Property with Boot Received (Of Cash)”

Gain/Loss Realized:

Amount realized

=

Fair market value of new auto + Boot received - Adjusted basis of auto given up

=

$16,500 fair market value new auto + $3,500 cash boot - $23,000 adjusted basis
of the old auto ($35,000 cost - $12,000 accumulated depreciation)

=

$3,000 loss

Gain/Loss Recognized:

Loss recognized

=

$0 (Realized loss is never recognized in like-kind exchanges.)

Basis of New Property:

New basis

=

Adjusted basis of property given up - Loss recognized - Boot received

=

$23,000 - $0 - $3,500

=

$19,500

Alternate calculation: $16,500 FMV new property + $3,000 deferred loss = $19,500 basis of new property.

Choice “c” is correct. $19,500 is the substituted basis of the new auto [$23,000 adjusted basis of the old auto ($35,000 cost - $12,000 accumulated depreciation) - $0 loss recognized - $3,500 boot received].

Choice “d” is incorrect. $16,500 is the fair market value of the new auto.

Choice “a” is incorrect. $23,000 is the adjusted basis of old auto ($35,000 cost - $12,000 accumulated depreciation).

Choice “b” is incorrect. $26,500 would be the new basis if the boot of $3,500 was paid, rather than received.

47
Q

A taxpayer is trading in an automobile used solely for business purposes for another automobile to be used in his business. The automobile originally cost $35,000 and he has taken $18,000 in depreciation. The old automobile is currently worth $20,000 and the new automobile the taxpayer wants in exchange is only worth $17,500. The other party agrees to give the taxpayer a trailer worth $3,500 in addition to the new auto, and the taxpayer agrees to pay $1,000 cash in addition to the trade-in. What is the gain or loss realized by the taxpayer on this transaction?

a.

$3,000 gain

b.

$2,500 gain

c.

$2,500 loss

d.

$500 gain

A

Calculations for “Realized Gain with Boot Received and Paid”

Gain/Loss Realized:

Amount realized

=

Fair market value of new auto + Boot received - Adjusted basis of auto given up

=

$17,500 fair market value new auto + $3,500 fair market value of trailer received -
$1,000 cash boot paid - $17,000 adjusted basis of the old auto ($35,000 cost -
$18,000 accumulated depreciation)

=

$3,000 gain

Gain/Loss Recognized:

Gain recognized

=

$3,000 (the lesser of realized gain of $3,000 or boot received of $3,500)

Basis of New Property:

New basis

=

Adjusted basis of property given up + Gain recognized - Boot received + Boot paid

=

$17,000 + $3,000 - $3,500 + $1,000

=

$17,500

Alternate calculation: $17,500 FMV new property + $0 deferred loss - $0 deferred gain = $17,500 basis new property.

Choice “a” is correct. A $3,000 gain is realized [fair market value of the new auto $17,500 + $3,500 fair market value of the trailer boot received - $1,000 cash boot paid - $17,000 adjusted basis of the old auto ($35,000 cost - $18,000 accumulated depreciation)].

Choices “b” and “c” are incorrect. $2,500 is the net value of the boots received and paid ($3,500 fair market value of the trailer received - $1,000 cash boot paid).

Choice “d” is incorrect. $500 would be the gain/loss realized if the boots paid and received were ignored.

48
Q

A taxpayer is trading in an automobile used solely for business purposes for another automobile to be used in his business. The automobile originally cost $35,000 and he has taken $18,000 in depreciation. The old automobile is currently worth $20,000 and the new automobile the taxpayer wants in exchange is only worth $17,500. The other party agrees to give the taxpayer a trailer worth $3,500 in addition to the new auto, and the taxpayer agrees to pay $1,000 cash in addition to the trade-in. What is the gain or loss recognized by the taxpayer in this transaction?

a.

$2,500 gain

b.

$3,500 gain

c.

$3,000 gain

d.

$0

A

Calculations for “Recognized Gain with Boot Received and Paid”

Gain/Loss Realized:

Amount realized

=

Fair market value of new auto + Boot received - Adjusted basis of auto given up

=

$17,500 fair market value new auto + $3,500 fair market value of trailer received -
$1,000 cash boot paid - $17,000 adjusted basis of the old auto ($35,000 cost -
$18,000 accumulated depreciation)

=

$3,000 gain

Gain/Loss Recognized:

Gain recognized

=

$3,000 (the lesser of realized gain of $3,000 or boot received of $3,500)

Basis of New Property:

New basis

=

Adjusted basis of property given up + Gain recognized - Boot received + Boot paid

=

$17,000 + $3,000 - $3,500 + $1,000

=

$17,500

Alternate calculation: $17,500 FMV new property + $0 deferred loss - $0 deferred gain = $17,500 basis of new property.

Choice “c” is correct. A $3,000 gain is recognized (the lesser of $3,000 gain realized and $3,500 fair market value of the trailer boot received).

Choice “b” is incorrect. $3,500 gain is the greater of the gain realized and boot received, not the lesser of the gain realized and boot received.

Choice “a” is incorrect. $2,500 is the difference between the fair market value of the trailer boot received, $3,500, and $1,000 cash boot paid.

Choice “d” is incorrect. Gain is recognized on like-kind transactions to the extent of boot received in like-kind transactions.

49
Q

A taxpayer is trading in an automobile used solely for business purposes for another automobile to be used in his business. The automobile originally cost $35,000 and he has taken $18,000 in depreciation. The old automobile is currently worth $20,000 and the new automobile the taxpayer wants in exchange is only worth $17,500. The other party agrees to give the taxpayer a trailer worth $3,500 in addition to the new auto, and the taxpayer agrees to pay $1,000 cash in addition to the trade-in. What is the taxpayer’s basis in the new automobile received?

a.

$14,500

b.

$17,500

c.

$19,500

d.

$17,000

A

Calculations for “New Basis on Like-Kind Property with Boot Received and Paid”

Gain/Loss Realized:

Amount realized

=

Fair market value of new auto + Boot received - Adjusted basis of auto given up

=

$17,500 fair market value new auto + $3,500 fair market value of trailer received -
$1,000 cash boot paid - $17,000 adjusted basis of the old auto ($35,000 cost -
$18,000 accumulated depreciation)

=

$3,000 gain

Gain/Loss Recognized:

Gain recognized

=

$3,000 (the lesser of realized gain of $3,000 or boot received of $3,500)

Basis of New Property:

New basis

=

Adjusted basis of property given up + Gain recognized - Boot received + Boot paid

=

$17,000 + $3,000 - $3,500 + $1,000

=

$17,500

Alternate calculation: $17,500 FMV new property + $0 deferred loss - $0 deferred gain = $17,500 basis of new property.

Choice “b” is correct. $17,500 is the substituted basis of the new auto [$17,000 adjusted basis of the old auto ($35,000 cost - $12,000 accumulated depreciation) + $3,000 gain recognized - $3,500 boot received + $1,000 boot paid].

Choice “a” is incorrect. A substituted basis of $14,500 ignores the $3,000 gain recognized.

Choice “d” is incorrect. $17,000 is the adjusted basis of the old auto.

Choice “c” is incorrect. A $19,500 basis adds the net value of the boots received and paid, rather than subtracts and also ignores the gain recognized.

50
Q

A taxpayer is trading in an automobile used solely for business purposes for another automobile to be used in his business. The automobile originally cost $35,000 and he has taken $18,000 in depreciation. The old automobile is currently worth $20,000 and the new automobile the taxpayer wants in exchange is worth $22,000. The taxpayer is also assuming a liability secured by the new auto of $2,000. What is the gain or loss realized by the taxpayer on this transaction?

a.

$3,000 gain

b.

$2,000 gain

c.

$0

d.

$5,000 gain

A

Calculations for “Realized Gain with Liabilities Assumed (Boot Paid)”

Gain/Loss Realized:

Amount realized

=

[Fair market value of auto received - Liabilities assumed (boot paid)] - Adjusted basis of auto given up

=

($22,000 fair market value new auto - $2,000 boot paid) - ($35,000 cost of old auto -
$18,000 accumulated depreciation)

=

$20,000 - $17,000

=

$3,000 gain

Gain/Loss Recognized:

Gain recognized

=

$0 (lesser of realized gain of $3,000 or boot received of $0)

Basis of New Property:

New basis

=

Adjusted basis of property given up + Liabilities assumed (boot paid)

=

$17,000 + $2,000

=

$19,000

Alternate calculation: $22,000 FMV new property - $3,000 deferred gain = $19,000 basis of new property.

Choice “a” is correct. A $3,000 gain is realized on the transaction [fair market value of the new auto, $22,000 - liabilities assumed (boot paid) of $2,000 - $17,000, the adjusted basis of the old auto ($35,000 cost - $18,000 accumulated depreciation)].

Choice “c” is incorrect. $0 is the gain/loss recognized (the lesser of $3,000 gain realized or boot received of $0), not the gain realized.

Choice “b” is incorrect. $2,000 is the difference between the fair market value of the old auto and the fair market value of the new auto.

Choice “d” is incorrect. $5,000 gain would be the gain realized if the liabilities assumed (boot paid) of $2,000 were ignored.

51
Q

A taxpayer is trading in an automobile used solely for business purposes for another automobile to be used in his business. The automobile originally cost $35,000 and he has taken $18,000 in depreciation. The old automobile is currently worth $20,000 and the new automobile the taxpayer wants in exchange is worth $22,000. The taxpayer is also assuming a liability secured by the new auto of $2,000. What is the gain or loss recognized by the taxpayer on this transaction?

a.

$0

b.

$5,000 gain

c.

$3,000 gain

d.

$2,000 gain

A

Calculations for “Recognized Gain with Liabilities Assumed (Boot Paid)”

Gain/Loss Realized:

Amount realized

=

[Fair market value of auto received - Liabilities assumed (boot paid)] - Adjusted basis of auto given up

=

($22,000 fair market value new auto - $2,000 boot paid) - ($35,000 cost of old auto -
$18,000 accumulated depreciation)

=

$20,000 - $17,000

=

$3,000 gain

Gain/Loss Recognized:

Gain recognized

=

$0 (lesser of realized gain of $3,000 or boot received of $0)

Basis of New Property:

New basis

=

Adjusted basis of property given up + Liabilities assumed (boot paid)

=

$17,000 + $2,000

=

$19,000

Alternate calculation: $22,000 FMV new property - $3,000 deferred gain = $19,000 basis of new property.

Choice “a” is correct. $0 of gain is recognized (the lesser of gain realized of $3,000 and boot received of $0).

Choice “d” is incorrect. $2,000 is the difference between the fair market value of the old auto and the fair market value of the new auto.

Choice “c” is incorrect. $3,000 is the gain realized, not the gain recognized.

Choice “b” is incorrect. A $5,000 gain would be realized if the boot paid of $2,000 was ignored. It still would not be the amount recognized.

52
Q

A taxpayer is trading in an automobile used solely for business purposes for another automobile to be used in his business. The automobile originally cost $35,000 and he has taken $18,000 in depreciation. The old automobile is currently worth $20,000 and the new automobile the taxpayer wants in exchange is worth $22,000. The taxpayer has agreed to assume a liability of $2,000 in addition to the trade-in. What is the taxpayer’s basis in the new automobile received?

a.

$19,000

b.

$22,000

c.

$15,000

d.

$17,000

A

Calculations for “New Basis of Like-Kind Property with Liabilities Assumed (Boot Paid)”

Gain/Loss Realized:

Amount realized

=

[Fair market value of auto received - Liabilities assumed (boot paid)] - Adjusted basis of auto given up

=

($22,000 fair market value new auto - $2,000 liability assumed) - ($35,000 cost of old auto - $18,000 accumulated depreciation)

=

$20,000 - $17,000

=

$3,000 gain

Gain/Loss Recognized:

Gain recognized

=

$0 (lesser of realized gain of $3,000 or boot received of $0)

Basis of New Property:

New basis

=

Adjusted basis of property given up + Liabilities assumed (boot paid)

=

$17,000 + $2,000

=

$19,000

Alternate calculation: $22,000 FMV new property - $3,000 deferred gain = $19,000 basis of new property.

Choice “a” is correct. $19,000 is the basis of the new auto [$17,000 adjusted basis of the old auto ($35,000 cost - $18,000 accumulated depreciation) + $2,000 in liabilities assumed (boot paid)].

Choice “c” is incorrect. A $15,000 basis would result from the subtraction of the $2,000 liability assumed (boot paid) from the $17,000 adjusted basis of the old auto, rather than from adding it.

Choice “d” is incorrect. $17,000 is the adjusted basis of the old auto ($35,000 cost - $18,000 accumulated depreciation). The liability assumed must also be included in the new basis.

Choice “b” is incorrect. $22,000 is the fair market value of new auto.

53
Q

A taxpayer is trading in an automobile used solely for business purposes for another automobile to be used in his business. The automobile originally cost $35,000 and he has taken $18,000 in depreciation. The old automobile is currently worth $20,000 and the new automobile the taxpayer wants in exchange is only worth $17,500. The other party also agrees to assume a liability from the taxpayer secured by the old auto of $2,500. What is the gain or loss realized by the taxpayer on this transaction?

a.

$3,000 gain

b.

$2,500 gain

c.

$500 gain

d.

$2,500 loss

A

Calculations for “Realized Gain with Liabilities Relieved (Boot Received)”

Gain/Loss Realized:

Amount realized

=

Fair market value of new auto + Liabilities relieved (boot received) - Adjusted basis of auto given up

=

$17,500 fair market value new auto + $2,500 relief from liability - $17,000 adjusted
basis of the old auto ($35,000 cost - $18,000 accumulated depreciation)

=

$3,000 gain

Gain/Loss Recognized:

Gain recognized

=

$2,500 [the lesser of realized gain of $3,000 or $2,500 in liabilities relieved (boot received)]

Basis of New Property:

New basis

=

Adjusted basis of property given up + Gain recognized - Liabilities relieved (boot received)

=

$17,000 + $2,500 - $2,500

=

$17,000

Alternate calculation: $17,500 FMV new property - $500 deferred gain = $17,000 basis of new property.

Choice “a” is correct. A $3,000 gain is realized on the transaction [fair market value of the new auto, $17,500 + $2,500 of liabilities relieved (boot received) - $17,000, the adjusted basis of the old auto ($35,000 cost - $18,000 accumulated depreciation)].

Choice “b” is incorrect. $2,500 is the amount of gain recognized, not realized.

Choice “c” is incorrect. A $500 gain would be the gain/loss realized if the $2,500 liability relieved (boot received) were ignored.

Choice “d” is incorrect. $2,500 is the difference between the fair market value of the old auto and the fair market value of the new auto.

54
Q

A taxpayer is trading in an automobile used solely for business purposes for another automobile to be used in his business. The automobile originally cost $35,000 and he has taken $18,000 in depreciation. The old automobile is currently worth $20,000 and the new automobile the taxpayer wants in exchange is only worth $17,500. The other party also agrees to assume a liability from the taxpayer secured by the old auto of $2,500. What is the gain or loss recognized by the taxpayer on this transaction?

a.

$500 gain.

b.

$3,000 gain.

c.

$0

d.

$2,500 gain.

A

Calculations for “Recognized Gain with Liabilities Relieved (Boot Received)”

Gain/Loss Realized:

Amount realized

=

Fair market value of new auto + Liabilities relieved (boot received) - Adjusted basis of auto given up

=

$17,500 fair market value new auto + $2,500 relief from liability - $17,000 adjusted
basis of the old auto ($35,000 cost - $18,000 accumulated depreciation)

=

$3,000 gain

Gain/Loss Recognized:

Gain recognized

=

$2,500 [the lesser of realized gain of $3,000 or $2,500 in liabilities relieved (boot received)]

Basis of New Property:

New basis

=

Adjusted basis of property given up + Gain recognized - Liabilities relieved (boot received)

=

$17,000 + $2,500 - $2,500

=

$17,000

Alternate calculation: $17,500 FMV new property - $500 deferred gain = $17,000 basis new property.

Choice “d” is correct. A gain of $2,500 is recognized on the transaction [the lesser of $3,000 gain realized and $2,500 in relief from the liability (boot received)].

Choice “b” is incorrect. $3,000 is the amount of gain realized, not recognized.

Choice “a” is incorrect. $500 would be the gain/loss realized if the relief from the liability (boot received) of $2,500 was ignored. It still would not be the amount recognized.

Choice “c” is incorrect. Gain is recognized to the extent of boot received (in this case, liabilities relieved) in like-kind transactions.

55
Q

A taxpayer is trading in an automobile used solely for business purposes for another automobile to be used in his business. The automobile originally cost $35,000 and he has taken $18,000 in depreciation. The old automobile is currently worth $20,000 and the new automobile the taxpayer wants in exchange is only worth $17,500. The other party also agrees to assume a liability from the taxpayer secured by the old auto of $2,500. What is the taxpayer’s basis in the new automobile received?

a.

$17,000

b.

$19,500

c.

$17,500

d.

$22,000

A

Calculations for “New Basis on Like-Kind Property with Liabilities Relieved (Boot Received)”

Gain/Loss Realized:

Amount realized

=

Fair market value of new auto + Liabilities relieved (boot received) - Adjusted basis of auto given up

=

$17,500 fair market value new auto + $2,500 relief from liability - $17,000 adjusted
basis of the old auto ($35,000 cost - $18,000 accumulated depreciation)

=

$3,000 gain

Gain/Loss Recognized:

Gain recognized

=

$2,500 [the lesser of realized gain of $3,000 or $2,500 in liabilities relieved (boot received)]

Basis of New Property:

New basis

=

Adjusted basis of property given up + Gain recognized - Liabilities relieved (boot received)

=

$17,000 + $2,500 - $2,500

=

$17,000

Alternate calculation: $17,500 FMV new property - $500 deferred gain = $17,000 basis new property.

Choice “a” is correct. The basis of the new auto is $17,000 [$17,000 adjusted basis of the old auto ($35,000 cost - $18,000 accumulated depreciation) + $2,500 gain recognized - $2,500 in liabilities relieved (boot received)].

Choice “c” is incorrect. $17,500 is the fair market value of the new auto.

Choice “b” is incorrect. A basis of $19,500 ignores the relieved liability.

Choice “d” is incorrect. A basis of $22,000 adds the $2,500 relief from liability (boot received) instead of subtracts it.

56
Q

A taxpayer is trading in an automobile used solely for business purposes for another automobile to be used in his business. The automobile originally cost $35,000 and he has taken $18,000 in depreciation. The old automobile is currently worth $20,000 and the new automobile the taxpayer wants in exchange is only worth $17,500. The taxpayer agrees to assume a liability secured by the new auto of $1,000. The other party also agrees to assume a liability secured by the taxpayer’s old auto of $3,500. What is the gain or loss realized by the taxpayer on this transaction?

a.

$500 gain

b.

$2,500 gain

c.

$2,000 loss

d.

$3,000 gain

A

Calculations for “Realized Gain with Liability Assumed (Boot Paid) and Liability Relieved (Boot Received)”

Gain/Loss Realized:

Amount realized

=

Fair market value of new auto + Boot received - Boot paid - Adjusted basis of auto given up

=

$17,500 fair market value new auto + $3,500 in relieved liabilities (boot received) -
$1,000 in liabilities assumed (boot paid) $17,000 adjusted basis of the old auto
($35,000 cost - $18,000 accumulated depreciation)

=

$3,000 gain

Gain/Loss Recognized:

Gain recognized

=

$2,500 [the lesser of realized gain of $3,000 or net relief from liabilities (boot received) of $2,500]

Basis of New Property:

New basis

=

Adjusted basis of property given up + Gain recognized - Boot received + Boot paid

=

$17,000 + $2,500 - $3,500 + $1,000

=

$17,000

Alternate calculation: $17,500 FMV new property - $500 deferred gain = $17,000 basis of new property.

Choice “d” is correct. A $3,000 gain is realized on the transaction [fair market value of the new auto $17,500 + $3,500 in liabilities relieved - $1,000 in liabilities assumed - $17,000 adjusted basis of the old auto ($35,000 cost - $18,000 accumulated depreciation)].

Choice “b” is incorrect. $2,500 is the gain recognized, not realized.

Choice “a” is incorrect. $500 would be the gain realized if the liabilities were ignored.

Choice “c” is incorrect. A $2,000 “loss” subtracts the $3,500 relief from liability (boot received) and adds the $1,000 liability assumed (boot paid) instead of adding the relief and subtracting the assumption of the liabilities.

57
Q

A taxpayer is trading in an automobile used solely for business purposes for another automobile to be used in his business. The automobile originally cost $35,000 and he has taken $18,000 in depreciation. The old automobile is currently worth $20,000 and the new automobile the taxpayer wants in exchange is only worth $17,500. The taxpayer agrees to assume a liability secured by the new auto of $1,000. The other party also agrees to assume a liability secured by the taxpayer’s old auto of $3,500. What is the gain or loss recognized by the taxpayer on this transaction?

a.

$3,500 gain.

b.

$0

c.

$3,000 gain.

d.

$2,500 gain.

A

Calculations for “Recognized Gain with Liability Assumed (Boot Paid) and Liability Relieved (Boot Received)”

Gain/Loss Realized:

Amount realized

=

Fair market value of new auto + Boot received - Boot paid - Adjusted basis of auto given up

=

$17,500 fair market value new auto + $3,500 in relieved liabilities (boot received) -
$1,000 in liabilities assumed (boot paid) $17,000 adjusted basis of the old auto
($35,000 cost - $18,000 accumulated depreciation)

=

$3,000 gain

Gain/Loss Recognized:

Gain recognized

=

$2,500 [the lesser of realized gain of $3,000 or net relief from liabilities (boot received) of $2,500]

Basis of New Property:

New basis

=

Adjusted basis of property given up + Gain recognized - Boot received + Boot paid

=

$17,000 + $2,500 - $3,500 + $1,000

=

$17,000

Alternate calculation: $17,500 FMV new property - $500 deferred gain = $17,000 basis of new property.

Choice “d” is correct. $2,500 in gain is recognized [the lesser of $3,000 gain realized and the $2,500 net liability relief. The net relief from liabilities (boot received) = $3,500 relief from liability (boot received) - $1,000 liability assumed (boot paid)].

Choice “a” is incorrect. $3,500 is the amount of liability relieved.

Choice “c” is incorrect. $3,000 is the gain realized, not recognized.

Choice “b” is incorrect. Gain is recognized to the extent of boot received in like-kind transactions.

58
Q

A taxpayer is trading in an automobile used solely for business purposes for another automobile to be used in his business. The automobile originally cost $35,000 and he has taken $18,000 in depreciation. The old automobile is currently worth $20,000 and the new automobile the taxpayer wants in exchange is only worth $17,500. The taxpayer agrees to assume a liability secured by the new auto of $1,000. The other party also agrees to assume a liability secured by the taxpayer’s old auto of $3,500. What is the taxpayer’s basis in the new automobile received?

a.

$17,500

b.

$19,500

c.

$17,000

d.

$14,500

A

Calculations for “New Basis on Like-Kind Property with Liability Assumed (Boot Paid) and Liability Relieved (Boot Received)”

Gain/Loss Realized:

Amount realized

=

Fair market value of new auto + Boot received - Boot paid - Adjusted basis of auto given up

=

$17,500 fair market value new auto + $3,500 in relieved liabilities (boot received) -
$1,000 in liabilities assumed (boot paid) $17,000 adjusted basis of the old auto
($35,000 cost - $18,000 accumulated depreciation)

=

$3,000 gain

Gain/Loss Recognized:

Gain recognized

=

$2,500 [the lesser of realized gain of $3,000 or net relief from liabilities (boot received) of $2,500]

Basis of New Property:

New basis

=

Adjusted basis of property given up + Gain recognized - Boot received + Boot paid

=

$17,000 + $2,500 - $3,500 + $1,000

=

$17,000

Alternate calculation: $17,500 FMV new property - $500 deferred gain = $17,000 basis of new property.

Choice “c” is correct. $17,000 is the basis of the new auto [$17,000 adjusted basis of the old auto ($35,000 cost - $18,000 accumulated depreciation) + $2,500 gain recognized - $2,500 net relief from liabilities (boot received) ($3,500 relief from liability (boot received) - $1,000 liability assumed (boot paid))].

Choice “d” is incorrect. A basis of $14,500 ignores the $2,500 gain recognized.

Choice “a” is incorrect. $17,500 is the fair market value of the new auto.

Choice “b” is incorrect. A basis of $19,500 adds the adjusted basis of the old property and the gain recognized, and ignores the boots paid and received.

59
Q

In the current year, Tatum exchanged farmland for an office building. The farmland had a basis of $250,000, a fair market value (FMV) of $400,000, and was encumbered by a $120,000 mortgage. The office building had an FMV of $350,000 and was encumbered by a $70,000 mortgage. Each party assumed the other’s mortgage. What is the amount of Tatum’s recognized gain?

a.

$150,000

b.

$0

c.

$50,000

d.

$100,000

A

Rule: Per IRC Section 1031, non-recognition treatment is accorded to a like-kind exchange of property used in a trade or business. “Like-kind” exchanges include exchanges of business property for business property, where like-kind is interpreted very broadly and refers to the nature or character of the property and not to its grade or quality.

Choice “c” is correct. The exchange in this question qualifies for Section 1031 treatment since the exchange appears to be business property for business property. However, the boot involved in the exchange (the mortgages) must be taken into account to determine the recognition or non-recognition of the gain realized on the exchange. In this transaction, the total consideration received by Tatum is the FMV of the property received of $350,000 plus the mortgage of $120,000 that was assumed by the other party, for a total of $470,000. The adjusted basis of the property given up was $250,000, and there is also $70,000 of mortgage given up by the other party (and assumed by Tatum), for a total of $320,000. The realized gain is thus $470,000 - $320,000 = $150,000. The recognized gain will be the lesser of realized gain or net boot received. The $120,000 of mortgage given up (and assumed by the other party) is treated as boot received, and the $70,000 of mortgage assumed is treated as boot given up. The net is $50,000 of boot received. The $50,000 of boot received is the recognized gain. The treatment is somewhat the same as if cash/boot had been received in the transaction.

Choice “b” is incorrect. Gain is recognized due to the net boot received.

Choice “d” is incorrect. The $100,000 is the amount of realized gain being deferred, not the recognized gain.

Choice “a” is incorrect. The $150,000 is the realized gain. However, it is not the recognized gain.

60
Q

Terry, a taxpayer, purchased stock for $12,000. Later, Terry sold the stock to a relative for $8,000. What amount is the relative’s gain or loss?

a.

$0.

b.

$4,000 gain.

c.

$2,000 loss.

d.

$2,000 gain.

A

Rule: IRC Section 267 controls the nonrecognition of realized losses on sales or exchanges of property to related parties. The most common related parties for individual taxpayers are members of a family (although there are certainly many other examples).

Choice “a” is correct. The loss realized on the transaction by Terry is $4,000 ($8,000 - $12,000). This transaction appears to qualify under Section 267. “Relative” is not defined in the question. Section 267 limits “family” to brothers and sisters, spouse, ancestor, and lineal descendants. However, the definition of relative is really irrelevant if the question is read closely. The question wants to know the relative’s gain or loss, not Terry’s gain or loss. Since all the relative did to this point was to buy the stock, the relative has no gain or loss.

Choices “c”, “d”, and “b” are incorrect, per the above explanation.

61
Q

Aviary Corp. sold a building for $600,000. Aviary received a down payment of $120,000 as well as annual principal payments of $120,000 for each of the subsequent four years. Aviary purchased the building for $500,000 and claimed depreciation of $80,000. What amount of gain should Aviary report in the year of sale using the installment method?

a.

$180,000

b.

$36,000

c.

$54,000

d.

$120,000

A

Rule: Under the installment method, revenue is reported over the period in which the cash payments are received. The amount of cash received is multiplied by the gross profit percentage on the sale to determine the revenue (which retains its character as capital gain or ordinary income, depending on the transaction).

Choice “b” is correct. The gross profit percentage is calculated as follows:

Sales Price

$600,000

Original Cost

$500,000

Accumulated Depreciation

(80,000)

Adjusted Basis

(420,000)

Realized Gain on Sale

$180,000

Gross Profit Percentage = $180,000 ÷ $600,000 = 30%

Gain Recognized in Year of Sale:$120,000 [cash received] × 30%= $36,000

Choice “a” is incorrect. The answer option recognizes as income the total realized gain ($180,000) on the sale. As indicated in the rule above, under the installment method, revenue is reported over the period in which the cash payments are received. The amount of cash received is multiplied by the gross profit percentage on the sale to determine the revenue.

Choice “d” is incorrect. This answer option is the total amount of cash received in the year of sale ($120,000). The gross profit percentage must be applied before the amount of revenue is determined. As indicated in the rule above, under the installment method, revenue is reported over the period in which the cash payments are received. The amount of cash received is multiplied by the gross profit percentage on the sale to determine the revenue.

Choice “c” is incorrect. This assumes a gross profit percentage of 45%. The actual gross profit percentage is 30%, as per the above calculation.

62
Q

Wynn, a single individual age 60, sold Wynn’s personal residence for $450,000. Wynn had owned Wynn’s residence, which had a basis of $250,000, for six years. Within eight months of the sale, Wynn purchased a new residence for $400,000. What is Wynn’s recognized gain from the sale of Wynn’s personal residence?

a.

$50,000

b.

$200,000

c.

$75,000

d.

$0

A

Rule: The sale of a taxpayer’s primary residence is subject to an exclusion from gross income for gain. A maximum of $250,000 gain exclusion is provided for all taxpayers other than married couples filing jointly. Married couples filing jointly have a maximum gain exclusion of $500,000. To qualify for the full exclusion, the taxpayers must have owned and used the property as a primary residence for two years or more during the five-year period ending on the date of the sale or exchange. There is no age requirement to receive the exclusion, and no roll-over to another house is required [these applied to an older tax law].

Choice “d” is correct. Wynn’s realized gain on the sale of the home is $200,000 [$450,000 - $250,000]. Wynn has owned and used the residence as his primary residence for the last six years. [Note that the purchase of the new home is of no consequence to the recognizable gain on the sale of the old home.] As the realized gain is less than the maximum excludable gain of $250,000 and Wynn has owned and used the property for more than two out of the last five years, Wynn has zero recognized gain on the sale of his residence.

Choice “a” is incorrect. This answer option incorrectly assumes that the gain is recognized to the extent the proceeds from the sale of the old house were not reinvested in the new house [$450,000 - $400,000 = $50,000]. Per the above rule, no roll-over to another house is required [these applied to an older tax law].

Choice “c” is incorrect. Wynn’s realized gain on the sale of the home is $200,000 [$450,000 - $250,000]. Wynn has owned and used the residence as his primary residence for the last six years. [Note that the purchase of the new home is of no consequence to the recognizable gain on the sale of the old home.] As the realized gain is less than the maximum excludable gain of $250,000 and Wynn has owned and used the property for more than two out of the last five years, Wynn has zero recognized gain on the sale of his residence.

Choice “b” is incorrect. Wynn’s realized gain on the sale of the home is $200,000 [$450,000 - $250,000], but the recognized gain is zero. Wynn has owned and used the residence as his primary residence for the last six years. As the realized gain is less than the maximum excludable gain of $250,000 and Wynn has owned and used the property for more than two out of the last five years, Wynn has zerorecognized gain on the sale of his residence.

63
Q

Sands purchased 100 shares of Eastern Corp. stock for $18,000 on April 1 of the prior year. On February 1 of the current year, Sands sold 50 shares of Eastern for $7,000. Fifteen days later, Sands purchased 25 shares of Eastern for $3,750. What is the amount of Sand’s recognized gain or loss?

a.

$500

b.

$0

c.

$2,000

d.

$1,000

A

Rule: A loss on a wash sale is disallowed for tax purposes. A wash sale exists when a security is sold for a loss and is repurchased within 30 days before or after the sale.

Choice “d” is correct. A wash sale exists in this case, but only a partial wash sale. Unfortunately, the dollar amounts for the recognized loss and wash sale (disallowed) loss are the same in this question (so the illustration can become somewhat confusing). Let’s use 20X1 and 20X2 for illustration. On 4/1/X1, Sands purchased 100s of Eastern stock for $18,000 ($180/share). On 2/1/X2, Sands sold 50s of the stock for $7,000 ($140/share), creating a realized loss of $2,000 (50s * ($140 - $180)). Now, if Sands had stopped there, it would have also had arecognized loss of $2,000. However, on 2/16/X2 Sands repurchased half of the shares it had sold at a loss (25s/50s), and this was within the 30-day period indicated in the rule (above). Thus, half of the realized loss is not recognizable in year 2, and it becomes part of the basis of the 25s of Eastern stock owned by Sands [note that the 50s not initially sold by Sands has a basis of $180/share, or $9,000]. The calculation follows:

2/1/X2

Sell 50s $7,000

Basis 50s (9,000) [$180 × 50 = $9,000, from the initial purchase]

Realized Loss $(2,000) [$40/share loss]

2/16/X2

Purchase 25s $3,750 [Repurchased within 30 days of loss sale]

Add: Wash Sale Loss 1,000 [($140 - $180) × 25s = $1,000]

Resulting Basis of 25s $4,750

Result: Sands owns 75s of Eastern stock. The first 50 shares (those that were not sold on 2/1/X2) has a basis of $9,000 in total ($180/share), and the 25 shares repurchased on 2/16/X2 has a basis of $4,750 ($190/share), the purchase price of the 25s plus the disallowed loss as a result of the wash sale rule.

Choice “b” is incorrect. Sands is allowed to recognize a loss on the shares it did not repurchase. The total loss on the 50 shares sold is $2,000, but Sands only repurchased 25 shares, so 50% of the $2,000 realized loss (or $1,000) is recognized by Sands in the year sold.

Choice “a” is incorrect. Sands is allowed to recognize a loss on the shares it did not repurchase. The total loss on the 50 shares sold is $2,000, but Sands only repurchased 25 shares, so 50% of the $2,000 realized loss (or $1,000) is recognized by Sands in the year sold. [Sands sold only 50% of the stock it initially purchased. Then, it repurchased 50% of the amount it sold.]

Choice “c” is incorrect. A wash sale exists in this situation. The entire $2,000 realized loss is not able to be recognized in the year sold because 25s of the 50s sold were repurchased in year 2. Only $1,000 of the realized loss is recognized in the year of sale. The remaining $1,000 loss is a wash sale loss that becomes part of basis (as shown above).

64
Q

Winkler, a CPA, provided accounting services to a client, Thompson. On December 15 of the same year, Thompson gave Winkler 100 shares of Foster Corp. as compensation for services. The adjusted basis of the stock was $4,000, and its fair market value at the time of transfer was $5,000. Two months later, Winkler sold the stock on February 15 for $7,500. What is the amount that Winkler should recognize as gain on the sale of stock?

a.

$0

b.

$5,000

c.

$2,500

d.

$1,000

A

Choice “c” is correct. The adjusted basis of the stock to Winkler was the $5,000 fair market value at the time of transfer (that same amount will be considered compensation in the form of property). The proceeds from the sale were $7,500. The gain on the sale of the stock was thus $2,500. The $4,000 adjusted basis of the stock to Thompson is irrelevant. Note that there is no “gift” here even though the word “gave” was used in the question.

Choice “a” is incorrect. There is gain on the sale, effectively in the amount of the increase in fair market value between the date of the transfer and the date of the sale.

Choice “d” is incorrect. The $1,000 appears to be the difference between Thompson’s basis and the fair market value of the stock on the date of the transfer to Winkler. This might be the gain to Thompson, but the question asks about Winkler.

Choice “b” is incorrect. The $5,000 fair market value of the stock on the date of the transfer is not the amount of the gain on the sale.

65
Q

On February 1, Year 4, Hall learned that he was bequeathed 500 shares of common stock under his father’s will. Hall’s father had paid $2,500 for the stock in Year 1. Fair market value of the stock on February 1, Year 4, the date of his father’s death, was $4,000 and had increased to $5,500 six months later. The executor of the estate elected the alternate valuation date for estate tax purposes. Hall sold the stock for $4,500 on June 1, Year 4, the date that the executor distributed the stock to him. How much income should Hall include in his Year 4 individual income tax return for the inheritance of the 500 shares of stock, which he received from his father’s estate? [Assume that the estate tax rules in effect for 2011 and forward apply].

a.

$0

b.

$2,500

c.

$4,000

d.

$5,500

A

Choice “a” is correct. There is no income tax on the value of inherited property. The gain on the sale is the difference between the sales price of $4,500 and Hall’s basis. Hall’s basis is the alternate valuation elected by the executor. This is the value six months after date of death or date distributed if before six months. The property was distributed four months after death and the value that day ($4,500) is used for the basis. $4,500 - $4,500 = 0.

Choice “d” is incorrect. There is no income tax on the value of inherited property.

Choice “c” is incorrect. This is the basis of the stock if the alternate date had not been used. Heirs are not taxed on inheritances. The income or loss results when inherited property is sold.

Choice “b” is incorrect. There is no income tax on the value of inherited property. The gain on the sale is the difference between the sales price of $4,500 and Hall’s basis. Hall’s basis is the alternate valuation elected by the executor.

66
Q

In the current year, Essex sold land with a basis of $80,000 to Yarrow for $100,000. Yarrow paid $25,000 down and agreed to pay $15,000 per year, plus interest, for the next five years, beginning in the second year. Under the installment method, what gain should Essex include in gross income for the year of sale?

a.

$20,000

b.

$5,000

c.

$25,000

d.

$15,000

A

Choice “b” is correct. Under the installment method, revenue is reported (recognized) over the period in which the cash payments are received. Included gross income is determined in 3 steps:

Step 1: Gross Profit:

Sale on Installment $ 100,000

Cost 80,000

Total Gross Profit $ 20,000

Step 2: Gross Profit Percentage:

Gross Profit/Sale on Installment ($20,000/$100,000) = 20%

Step 3: Taxable Gross Profit:

Collections ($25,000) x Gross Profit Percentage (20%) = $5,000

67
Q

Hogan exchanged a business-use machine having an original cost of $100,000 and accumulated depreciation of $30,000 for business-use equipment owned by Baker having a fair market value of $80,000 plus $1,000 cash. Baker assumed a $2,000 outstanding debt on the machine. What taxable gain should Hogan recognize?

a.

$3,000

b.

$11,000

c.

$0

d.

$10,000

A

Choice “a” is correct. In a like-kind exchange, if property other than property qualifying for such exchange is received, (e.g., cash known as “boot”), the transaction, while not qualified for complete nonrecognition, produces recognized gain. The recognized gain is the lower of realized gain or the boot. Cancellation of debt is classified as “boot,” so the total boot is $3,000 ($1,000 cash + $2,000 debt cancellation).

Choice “c” is incorrect. To avoid all immediate gain on a like-kind exchange, boot cannot be received.

Choice “d” is incorrect. The exchange of the like-kind assets alone does not result in gain.

Choice “b” is incorrect. The exchange of the like-kind assets alone does not result in gain.

68
Q

A married couple purchased their principal residence for $300,000. They spent $40,000 on improvements. After living in it for 10 years, the couple sold the home for $650,000 and paid $36,000 in real estate commissions. What gain should the couple recognize on their joint return?

a.

$310,000

b.

$274,000

c.

$60,000

d.

$0

A

Choice “d” is correct. The sale of the taxpayer’s personal (primary or principal) residence is subject to an exclusion from gross income for gain of $500,000 married filing joint or $250,000 single. To qualify, the taxpayer must have owned and used the property as a principal residence for two years or more during the five year period ending on the date of the sale or exchange.

Taxpayer’s Basis:

$ 300,000

Purchase price

40,000

Improvements

36,000

Real estate commissions

376,000

Ending basis

Sales Price:

650,000

Gain on sale:

$ 274,000

Under allowed $500,000 exclusion for married couple

Choice “c” is incorrect based on the above calculation.

Choice “b” is incorrect. $274,000 is the realized gain, yet it does not need to berecognized.

Choice “a” is incorrect based on the above calculation.

69
Q

Judy and Kevin Kales had the following stock sales during the current taxable year:

Gross
Proceeds Basis

Crispy Crunch, Inc. $ 4,000 $ 5,000

Summer Solstice, Inc. 3,500 3,000

Sealy & Sealy, Inc. 2,000 10,000

Each stock was held for over 12 months. What amount should be reported on their current year tax return for capital gain/loss?

a.

$3,000 loss

b.

$0

c.

$500 income

d.

$8,500 loss

A

Choice “a” is correct. The Kales’ transactions net to a $8,500 loss on the sales. The current year transactions should be netted first, then additional amounts up to $3,000 can be used to offset ordinary income.

70
Q

Marsha and Brad, married taxpayers filing jointly, had the following transactions during Year 9:

Gain on sale of stock purchased in Year 1 and sold in June, Year 9 $ 3,000

Ordinary income from employers 80,000

Loss on sale of stock purchased in January, Year 9 and sold in March, Year 9 20,000

What is the amount of the capital loss carryover to Year 10?

a.

$14,000

b.

$17,000

c.

$20,000

d.

$0

A

Choice “a” is correct. In Year 9, Marsha and Brad had a net capital loss of $17,000, of which an additional $3,000 can be used to offset income from other sources (for example, the ordinary income from employment) in the current year. This would reduce the carryforward to $14,000.

Choice “d” is incorrect. Taxpayers are limited to a maximum capital loss of $3,000 offsetting income from other sources (after offsetting any appropriate capital gains). Therefore, the net total $17,000 loss on the sale of the stock cannot be used in the year it is incurred.

Choice “c” is incorrect. Capital losses can offset any gains in the year incurred, and then a maximum of $3,000 of income from other sources can be offset with capital losses.

Choice “b” is incorrect. Choice “b” is the net of the current year loss and gain; however, an additional $3,000 of the loss can be recognized in the current year.

71
Q

David Dickens sold a piece of land on an installment sale basis to Chuck Quarters on October 1, Year 1. David’s basis in the land was $150,000 and Chuck purchased it for $1,500,000; payable in three annual installments of $500,000 on Dec. 31, Year 1, Year 2 and Year 3. How much gain should David report in his Year 1 tax return?

a.

$500,000

b.

$1,500,000

c.

$450,000

d.

$1,350,000

A

Choice “c” is correct. The realized and recognized profit should be calculated as follows:

Step 1: Gross Profit

Sale on installment $ 1,500,000

Cost of goods sold (basis) 150,000

Total gross profit $ 1,350,000

Step 2: Gross Profit Percentage

Gross profit
Sales on installment

$1,350,000
$1,500,000

= 90%

Step 3: Earned Gross Profit

Current year payment: $500,000 x 90% = $450,000

72
Q

Chris and Jennifer purchased their home in California on January 15, Year 1, for $160,000. During their ownership they made no capital improvements. On August 1, Year 4, the couple moved to Virginia from California and rented out that home. On June 30, Year 6; the couple contracted to sell the California rental home for $437,500. For the calendar Year 6, the couple will file a joint tax return. Disregarding any depreciation recapture rules, how should they treat the sale of the home for tax purposes?

a.

Realized and recognized gain of $277,500; taxable on Schedule E.

b.

Realized and recognized gain of $277,500, taxable on Schedule D.

c.

Realized gain of $437,500; not taxable due to the home exclusion.

d.

Realized gain of $277,500; not taxable due to the home exclusion.

A

Choice “d” is correct. Disregarding any depreciation recapture, Chris and Jennifer have a realized gain of $277,500. For tax purposes, this gain will not be recognized on their Year 6 tax return as it is excludable under the Homeowner’s Exclusion. To qualify for the full exclusion of $500,000 for a joint return, the taxpayers must own and use the home as the principal residence for two years out of the five- year period ending on the date of the sale or exchange (and may not have any unqualified use after 2008).

Choices “b” and “a” are incorrect. In both these cases, the gain would not be recognized and taxed.

Choice “c” is incorrect. The realized gain is the difference between the taxpayer’s basis and the gross proceeds from the sale; not just the gross proceeds.

73
Q

On February 1, year 1, a taxpayer purchased an option to buy 1,000 shares of XYZ Co. for $200 per share. The taxpayer purchased the option for $50,000, which was to remain in effect for six months. The market declined, and the taxpayer let the option lapse on August 1, year 1. The taxpayer would report which of the following as a capital loss on the year 1 income tax return?

a.

$150,000 long term.

b.

$50,000 long term.

c.

$200,000 short term.

d.

$50,000 short term.

A

Choice “d” is correct. An option held by an investor is a capital asset. A capital asset which is sold or exchanged within one year of acquisition will generate either a short-term capital gain (if the capital asset is sold at a price greater than acquisition cost) or a short-term capital loss (if the capital asset is sold at a price less than the acquisition cost). The cost (or other basis) of worthless stock or securities is treated as a capital loss as if they were sold on the last day of the taxable year in which they became totally worthless. The option’s exercise price is irrelevant with respect to determining loss on account of the lapse of the options.

In this question, the options, which were capital assets purchased for $50,000 on February 1, Year 1, became worthless on the lapse date, August 1, Year 1. Thus, the $50,000 capital loss is treated as having occurred on December 31, Year 1, the last day of the taxable year in which the options became totally worthless. Because, as of December 31, Year 1, the options had not been held for more than a year, the $50,000 capital loss will be reported on the income tax return as a short-term capital loss.

Choices “b”, “a”, and “c” are incorrect per the above rules.

74
Q

A taxpayer lived in an apartment building and had a two-year lease that began 16 months ago. The taxpayer’s landlord wanted to sell the building and offered the taxpayer $10,000 to vacate the apartment immediately. The taxpayer’s lease on the apartment was a capital asset but had no tax basis. If the taxpayer accepted the landlord’s offer, the gain or loss would be which of the following?

a.

An ordinary gain.

b.

A long-term capital gain.

c.

A short-term capital loss.

d.

A short-term capital gain.

A

Choice “b” is correct. A capital asset which is sold or exchanged more than one year after the date of acquisition will generate either a long-term capital gain (if the capital asset is sold at a price greater than acquisition cost) or a long-term capital loss (if the capital asset is sold at a price less than the acquisition cost). In this question, the lease-hold interest, which is a capital asset, was acquired more than a year ago, and the basis (acquisition cost) in that capital asset is -0-. So, the receipt of $10,000 to vacate the apartment will generate a $10,000 long-term capital gain.

Choices “a”, “c”, and “d” are incorrect per the above rules.

75
Q

In year 1, a taxpayer sold real property for $200,000, receiving $100,000 at closing and $100,000 plus accrued interest at the prime rate in the next year. The buyer also assumed a $50,000 mortgage on the property. The taxpayer’s adjusted basis was $75,000, and the taxpayer incurred $10,000 of selling expenses. If this transaction qualifies for installment sale treatment, what is the gross profit on the sale?

a.

$165,000

b.

$125,000

c.

$175,000

d.

$115,000

A

Choice “a” is correct. Unless the taxpayer elects not to use the installment sales method, the taxpayer generally will recognize gain (but not loss) over the period during which the taxpayer receives cash payments (other than interest income) from the sale of noninventory assets. Note that this method is not available for the sale of stocks and securities traded on an established securities market.

The gross profit will be the amount realized less selling costs less the adjusted basis of the property sold (note: IRS forms require the taxpayer (i) to increase the adjusted basis by the amount of the selling costs and (ii) not reduce the amount realized by the selling costs. This requirement does not change the amount of gain/gross profit.

If the contract requires that payments be made in a subsequent year and if the contract requires little or no interest, the taxpayer may have to reduce the amount realized by the amount of unstated interest. This rule does not apply here because the contract requires that the buyer pay accrued interest at the prime rate in the next year.

Amount realized:

Cash to be received, excluding interest income

$ 200,000

Related debt assumed by the buyer

50,000

Less: selling expenses

(10,000)

Amount realized

$ 240,000

Less: Adjusted basis

(75,000)

Gain realized/gross profit

$ 165,000

Choices “d”, “b”, and “c” are incorrect per the above rule and per the above computations.

76
Q

Upon her grandfather’s death, Jordan inherited 10 shares of Universal Corp. stock that had a fair market value of $5,000. Her grandfather acquired the shares in 1995 for $2,500. Four months after her grandfather’s death, Jordan sold all her shares of Universal for $7,500. What was Jordan’s recognized gain in the year of sale?

a.

$2,500 long-term capital gain.

b.

$5,000 short-term capital gain.

c.

$2,500 short-term capital gain.

d.

$5,000 long-term capital gain.

A

Choice “a” is correct. Unless the executor elects the “alternative valuation date” method (not applicable to this question), the basis of property acquired by bequest or by inheritance is the property’s fair market value on the date of the decedent’s death. The decedent’s basis is irrelevant. Additionally, such acquired property is always considered to be “long-term” property, regardless of how long it has been held by the decedent and by the beneficiary or heir.

Calculation of gain realized and recognized:

Amount realized

$ 7,500

Less: Basis (date-of-death fair market value)

(5,000)

Long-term capital gain realized and recognized

$ 2,500

Choices “c”, “d”, and “b” are incorrect per the above rule.

77
Q

For an individual business owner, which of the following would typically be classified as a capital asset for federal income tax purposes?

a.

Accounts receivable.

b.

Inventory.

c.

Machinery and equipment used in a business.

d.

Marketable securities.

A

Choice “d” is correct. Capital assets include all marketable securities unless the taxpayer is a dealer.

Choice “a” is incorrect. Accounts receivable of a business are excluded from the definition of capital assets.

Choice “c” is incorrect. All depreciable assets of a business are excluded from the definition of capital assets. They are defined as Section 1231 assets.

Choice “b” is incorrect. Inventory of a business is excluded from the definition of capital assets.

78
Q

Simmons gives her child a gift of publicly-traded stock with a basis of $40,000 and a fair market value of $30,000. No gift tax is paid. The child subsequently sells the stock for $36,000. What is the child’s recognized gain or loss, if any?

a.

$4,000 loss.

b.

$6,000 gain.

c.

No gain or loss.

d.

$36,000 gain.

A

Choice “c” is correct. This situation falls into the exception of the gift tax basis rule because the FMV at date of gift is lower than the donor’s original basis. The donee then sold the stock at a price less than the donor’s rollover cost basis but higher than the FMV on date of gift. Therefore, there is no gain or loss on the sale.

Choices “a”, “b”, and “d” are incorrect per the above explanation.

79
Q

An individual entered into several exchanges during the current tax year. Which of the following exchanges is classified as like-kind?

a.

Apartment building for unimproved land.

b.

Common stock for common stock.

c.

Manufacturing equipment for factory building.

d.

Partnership interest for partnership interest.

A

Choice “a” is correct. Real property exchanged for other real property will be classified as a like-kind exchange (unless the property is in different countries).

Choice “d” is incorrect. Partnership interests are specifically excluded from like-kind exchange classification.

Choice “b” is incorrect. Common stock is specifically excluded from like-kind exchange classification.

Choice “c” is incorrect. Manufacturing equipment is not like-kind to a factory building. The factory building is real property and is only like-kind to other real property.

80
Q

Baker, an unmarried individual, sold a personal residence, which has an adjusted basis of $70,000, for $165,000. Baker owned and lived in the residence for seven years. Selling expenses were $10,000. Four weeks prior to the sale, Baker paid a handyman $1,000 to paint and fix up the residence. What is the amount of Baker’s recognized gain?

a.

$85,000

b.

$0

c.

$95,000

d.

$84,000

A

Choice “b” is correct. This is a principal residence that the taxpayer has owned and lived in for the last seven years. This exceeds the requirement of at least two of the last five years. Baker may therefore exclude up to $250,000 of gain. The realized gain is $84,000 ($165,000 selling price – $70,000 adjusted basis – $10,000 selling expenses – $1,000 fix-up expenses incurred within 90 days of the sale). All of the realized gain is excluded, and none of it is recognized.

Choice “d” is incorrect. $84,000 is the correct realized gain. But none of the gain is recognized, due to the homeowner’s exclusion.

Choice “a” is incorrect. $85,000 is the realized gain before considering the fix-up expenses of $1,000. But none of the gain is recognized, due to the homeowner’s exclusion.

Choice “c” is incorrect. $95,000 is the realized gain before considering the selling expenses and fix-up expenses. But none of the gain is recognized, due to the homeowner’s exclusion

81
Q

An individual reports the following capital transactions in the current year:

Short-term capital gain

$ 1,000

Short-term capital loss

$ (11,000)

Long-term capital gain

$ 10,000

Long-term capital loss

$ (6,000)

What amount is deducted in arriving at adjusted gross income?

a.

$10,000

b.

$3,000

c.

$6,000

d.

$0

A

Choice “b” is correct. First, the long-term capital gains and losses are netted to arrive at a net long-term capital gain of $4,000. Next, the short-term capital gains and losses are netted to arrive at a net short-term capital loss of $10,000. The next step is to net the net long-term capital gain of $4,000 with the net short-term capital loss of $10,000. This results in a net capital loss of $6,000. Only $3,000 of that loss is currently deductible against ordinary income. The remaining loss of $3,000 is carried forward indefinitely.

Choice “a” is incorrect. $10,000 is the net short-term capital loss before considering any of the long-term items.

Choice “c” is incorrect. $6,000 is the net capital loss, but the deduction is limited to $3,000.

Choice “d” is incorrect. The rules do allow for a net capital loss of up to $3,000 to be deducted against ordinary income.

82
Q

On March 1 of the previous year, a parent sold stock with a cost of $8,000 to their child, for $6,000, its fair market value. On September 30 of the current year, the child sold the same stock for $7,000 to Hancock, who is unrelated to the parent and child. What is the proper treatment for these transactions?

a.

Parent has $0 recognized loss and child has $1,000 recognized gain.

b.

Parent has $2,000 recognized loss and child has $0 recognized gain.

c.

Parent has $0 recognized loss and child has $0 recognized gain.

d.

Parent has a $2,000 recognized loss and child has $1,000 recognized gain.

A

Choice “c” is correct. The parent has a realized loss of $2,000 ($6,000 sale less $8,000 cost). However, none of this loss is recognized, because it is disallowed under the related party transaction rules. The child has a realized gain of $1,000 ($7,000 sale less $6,000 cost). This gain can be reduced (but not below zero) by the disallowed loss of the parent. Therefore, the recognized gain to the child is zero.

Choices “d”, “b”, and “a” are incorrect, per the above rule.

83
Q

A sole proprietor of a farm implement store sold a truck for $15,000 that had been used to make service calls. The truck cost $30,000 three years ago, and $21,360 depreciation was taken. What is the appropriate classification of the $6,360 gain for tax purposes?

a.

Section 1231 (Property Used in the Trade or Business and Involuntary Conversions) gain.

b.

Long-term capital gain.

c.

Short-term capital gain.

d.

Ordinary gain.

A

Choice “d” is correct. The truck is a depreciable asset used in a trade or business. Therefore, it is a Section 1231 asset. It is also personal property, so the recapture rules of Section 1245 will apply to any gains. The truck was sold at a gain. However, that gain is less than the accumulated depreciation. Under the rules of Section 1245, the gain is all recaptured as an ordinary gain.

Choices “a”, “b”, and “c” are incorrect, per the above rule.

84
Q

Dove Corp. began operating a hardware store in the current year after constructing a building at a total cost of $100,000 on land previously acquired for $50,000. In the current year, the land had a fair market value of $60,000. Dove paid real estate taxes of $5,000 in the current year. What is the total depreciable basis of Dove’s business property?

a.

$100,000

b.

$150,000

c.

$155,000

d.

$160,000

A

Choice “a” is correct. The only amount that may be depreciated is the $100,000 that Dove spent to construct the building. The $50,000 cost of the land is not depreciable as land is not a depreciable asset. The fair market value of the land ($60,000) is irrelevant for depreciation purposes. The real estate taxes ($5,000) are a deductible expense to the business that would not be capitalized.

Choice “b” is incorrect. The cost of the land is not a depreciable expense.

Choice “c” is incorrect. The cost of the land is not a depreciable expense. The real estate taxes ($5,000) are a deductible expense to the business that would not be capitalized.

Choice “d” is incorrect. The fair market value of the land ($60,000) is irrelevant for depreciation purposes.

85
Q

Rock Crab, Inc. purchases the following assets during the year:

Computer $ 3,000

Computer desk 1,000

Office furniture 4,000

Delivery van 25,000

What should be reported as the cost basis for MACRS five-year property?

a.

$3,000

b.

$33,000

c.

$28,000

d.

$25,000

A

Choice “c” is correct. MACRS 5-year property includes automobiles, light trucks, computers, typewriters, copiers, duplicating equipment, and other such items. The cost basis of the MACRS 5-year property is $28,000, calculated as follows:

Computer $ 3,000

Delivery van 25,000

MACRS 5-year $ 28,000

Choice “a” is incorrect. While the computer ($3,000) is included, the delivery van ($25,000) is also 5-year MACRS property.

Choice “d” is incorrect. While the delivery van ($25,000) is included, the computer ($3,000) is also MACRS 5-year property.

Choice “b” is incorrect. This answer option assumes that all of the assets in the question are MACRS 5-year property. However, the computer desk and the office furniture are MACRS 7-year property, which includes office furniture and fixtures, equipment and property with no ADR midpoint classified elsewhere, and railroad track.

86
Q

Decker sold equipment for $200,000. The equipment was purchased for $160,000 and had accumulated depreciation of $60,000. What amount is reported as ordinary income under Code Sec. 1245?

a.

$60,000

b.

$0

c.

$40,000

d.

$100,000

A

Choice “a” is correct. Under Sec. 1245, ordinary income is recognized on the gain to the extent of the accumulated depreciation. Any gain in excess of the original cost is capital gain.

Choice “b” is incorrect. The total gain is $100,000 which is the sales price ($200,000) in excess of cost ($160,000) less depreciation ($60,000). Under Sec. 1245, ordinary income is recognized on the gain to the extent of the accumulated depreciation. Any gain in excess of the original cost is capital gain.

Choice “c” is incorrect. Under Sec. 1245, ordinary income is recognized on the gain to the extent of the accumulated depreciation. Any gain in excess of the original cost is capital gain. The total gain is $100,000, but only $60,000 is ordinary income and $40,000 is capital gain income.

Choice “d” is incorrect. The total gain is $100,000, but only $60,000 is ordinary income and $40,000 is capital gain income.

87
Q

Lobster, Inc. incurs the following losses on disposition of business assets during the year:

Loss on the abandonment of office equipment

$25,000

Loss on the sale of a building (straight-line
depreciation taken in prior years of $200,000)

250,000

Loss on the sale of delivery trucks

15,000

What is the amount and character of the losses to be reported on Lobster’s tax return?

a.

$40,000 Section 1231 loss only.

b.

$40,000 Section 1231 loss, $250,000 long-term capital loss.

c.

$290,000 Section 1231 loss.

d.

$40,000 Section 1231 loss, $50,000 long-term capital loss.

A

Choice “c” is correct. Section 1231 assets are comprised principally of depreciable personal and real property used in the taxpayer’s trade or business and held for over twelve months. Trade or business property and capital assets (held over twelve months) that have been involuntarily converted are also included. All of the assets listed in this problem are Section 1231 assets. Net 1231 losses (Sec. 1231 losses less Sec. 1231 gains) are treated as ordinary losses.

88
Q

Which of the following conditions must be satisfied for a taxpayer to expense, in the year of purchase, under Internal Revenue Code Section 179, the cost of new or used tangible depreciable personal property?

I.

The property must be purchased for use in the taxpayer’s active trade or business.

II.

The property must be purchased from an unrelated party.

a.

Both I and II.

b.

Neither I nor II.

c.

II only.

d.

I only.

A

Choice “a” is correct. To qualify for IRC Section 179, the property must be tangible personal property acquired by purchase from an unrelated party for use in the active conduct of a trade or business.

Statements I and II are both correct statements concerning the criteria for property to qualify under IRC Section 179.

89
Q

A taxpayer purchased five acres of land for $20,000 and placed in service other tangible business assets that cost $100,000. Disregarding business income limitations and assuming that the annual Section 179 (Election to Expense Certain Depreciable Business Assets) limit is $108,000, what maximum amount of cost recovery can the taxpayer claim this year?

a.

$120,000

b.

$108,000

c.

$20,000

d.

$100,000

A

Choice “d” is correct. Under the election to expense certain depreciable business assets (sec. 179), the taxpayer may expense the cost of the depreciable asset up to the limitation, in this example $108,000. Therefore, only the cost of the depreciable tangible business assets can be expensed ($100,000).

Choice “a” is incorrect. Land is not a depreciable asset.

Choice “b” is incorrect. Taxpayer can only expense up to the purchase price, not to exceed the limitation.

Choice “c” is incorrect. Land is not a depreciable asset.

90
Q

Bent Corp., a calendar-year C corporation, purchased and placed into service residential real property during February, Year 8. No other property was placed into service during Year 8. What convention must Bent use to determine the depreciation deduction for the alternative minimum tax?

a.

Mid-quarter.

b.

Mid-month.

c.

Full-year.

d.

Half-year.

A

Choice “b” is correct. Real property (buildings) is subject to the mid-month convention under MACRS. Only personal property (machinery & equipment) is subject to the half-year and/or mid-quarter conventions.

91
Q

Data Corp., a calendar year corporation, purchased and placed into service office equipment during November Year 1. No other equipment was placed into service during Year 1. Under the general MACRS depreciation system, what convention must Data use?

a.

Mid-month.

b.

Half-year.

c.

Full-year.

d.

Mid-quarter.

A

Choice “d” is correct. When a taxpayer places 40% or more of its property (other than certain qualifying real property) into service in the last quarter of the taxable year, the corporation must use the mid-quarter convention for MACRS depreciation purposes. With this method the acquisitions are segregated by quarter and treated as if placed in service in the middle of each respective quarter.

92
Q

On August 1, Year 1, Graham purchased and placed into service an office building costing $264,000 including $30,000 for the land. What was Graham’s MACRS deduction for the office building in Year 1?

a.

$9,600

b.

$3,600

c.

$2,250

d.

$6,000

A

Choice “c” is correct. Only the building is depreciable, so the depreciable portion is $264,000 less $30,000 land, for a net of $234,000. The MACRS rules provide a 39-year life, straight-line depreciation, and a “mid-month” acquisition convention that treats the property as acquired in the middle of the month, regardless of the actual date of acquisition. Therefore, the August 1, Year 1, service date provides a half-month’s depreciation for August, plus a full month for September through December, for a total of 4.5 months for Year 1. ($234,000/39 years) × (4.5/12) = $2,250.

Choice “a” is incorrect. The recovery period for nonresidential real property is 39 years and the mid-month convention is used. The building is treated as if acquired in the middle of the month, regardless of the actual date of acquisition. Depreciation may only be taken for the months after the building was placed in service.

Choice “d” is incorrect. The mid-month convention is used for real property. The building is treated as if acquired in the middle of the month, regardless of the actual date of acquisition. Depreciation may only be taken for the months after the building was placed in service.

Choice “b” is incorrect. The recovery period for nonresidential real property is 39 years.

93
Q

How is the depreciation deduction of nonresidential real property determined for regular tax purposes using MACRS?

a.

150% declining-balance method with a switch to the straight-line method over 39 years.

b.

150% declining-balance method with a switch to the straight-line method over 27.5 years.

c.

Straight-line method over 40 years.

d.

Straight-line method over 39 years.

A

Choice “d” is correct. Nonresidential realty is depreciated over 39 years straight-line if placed in service after May 1993

94
Q

Platt owns land that is operated as a parking lot. A shed was erected on the lot for the related transactions with customers. With regard to capital assets and Section 1231 assets, how should these assets be classified?

~Land
~Shed
a.

Section 1231

Section 1231

b.

Capital

Capital

c.

Section 1231

Capital

d.

Capital

Section 1231

A

Choice “a” is correct. Because the parking lot and the shed constitute real estate and depreciable assets used in a trade or business, they are not capital assets per the definition below.

Note: The parking lot and shed will fall under Section 1231 (provided they are used in the business over 12 months) and possibly Section 1250 and 1245, respectively, upon sale of the assets.

Capital assets are defined as all property held by the taxpayer, except:

Property normally included in inventory or held for sale to customers in the ordinary course of business.

Depreciable property and real estate used in business.

Accounts and notes receivable arising from sales or services in the taxpayer’s business.

Copyrights, literary, musical or artistic compositions.

Treasury stock.

95
Q

In Year 6, an IRS agent completed an examination of a corporation’s Year 5 tax return and proposed an adjustment that will result in an increase in taxable income for each of Years 1 through 5. All returns were filed on the original due date. The proposed adjustment relates to the disallowance of corporate jet usage for personal reasons. The agent does not find the error to be fraudulent or substantial in nature.

Which of the following statements regarding this adjustment is correct?

a.

The adjustment is proper because it relates to a change in accounting method, which can be made retroactively irrespective of the statute of limitations.

b.

The adjustment is improper because an agent may only propose adjustments to the year under examination.

c.

The adjustment is improper because the statute of limitations has expired for several years of the adjustment.

d.

The adjustment is proper because there is no statute of limitations for improperly claiming personal expenses as business expenses.

A

Choice “c” is correct. Unless there is a substantial 25% misstatement of income or fraud, the statue of limitations is generally three years from the later of the due date or filing date of a return. This adjustment is improper because there is no evidence of fraud or substantial misstatement and some of the years are old enough that the three year statute has expired.

Choice “b” is incorrect. The adjustment is improper, but not because only the current year can be adjusted. Generally, the statute of limitations is for three years, and any of those “open” years can be adjusted. The three year statute of limitations has expired for some of the years in this question.

Choice “d” is incorrect. It is not correct that there is no statute of limitations for claiming personal expenses as business expenses.

Choice “a” is incorrect. It is not correct that this retroactive type of adjustment can circumvent the statute of limitations

96
Q

The sale of which of the following types of business property should be reported as Section 1231 (Property Used in the Trade or Business and Involuntary Conversions) property?

a.

Cattle held for 6 months.

b.

Machinery held for six months.

c.

Land held for 18 months.

d.

Inventory held for resale.

A

Choice “c” is correct. 1231 Assets are depreciable personal property and real property used in a business and held for over 12 months. Land held for 18 months meets this definition.

Choice “d” is incorrect. 1231 Assets are depreciable personal property and real property used in a business and held for over 12 months. Inventory held for resale does not meet this definition.

Choice “b” is incorrect. 1231 Assets are depreciable personal property and real property used in a business and held for over 12 months. Machinery held for six months does not meet this definition.

Choice “a” is incorrect. 1231 Assets are depreciable personal property and real property used in a business and held for over 12 months. Cattle held for six months does not meet this definition.

97
Q

A taxpayer sold for $200,000 equipment that had an adjusted basis of $180,000. Through the date of the sale, the taxpayer had deducted $30,000 of depreciation. Of this amount, $17,000 was in excess of straight-line depreciation. What amount of gain would be recaptured under Section 1245 (Gain from Dispositions of Certain Depreciable Property)?

a.

$30,000

b.

$17,000

c.

$13,000

d.

$20,000

A

Choice “d” is correct. Under Section 1245, the amount of depreciation in excess of straight-line depreciation is irrelevant (the “excess depreciation” rule is a Section 1250 rule and applies to real property). In this question, $30,000 of depreciation was deducted and, at first glance, the answer would appear to be $30,000. However, Section 1245 actually only requires that the lesser of the depreciation taken ($30,000) or the gain recognized ($200,000 - $180,000 = $20,000) be recaptured.

Choice “c” is incorrect. The $13,000 is the difference between the $30,000 depreciation that had been deducted and the $17,000 “excess” depreciation. That amount is not used anywhere.

Choice “b” is incorrect. The $17,000 is the amount of “excess” depreciation. As indicated, this amount is related to Section 1250 depreciation recapture, not Section 1245. Section 1245 requires depreciation recapture regardless of the method of depreciation.

Choice “a” is incorrect. The $30,000 is the depreciation that had been deducted. Normally, that would be the amount subject to Section 1245 recapture. However, only the gain recognized is recaptured in this question.

98
Q

Four years ago, a self-employed taxpayer purchased office furniture for $30,000. During the current tax year, the taxpayer sold the furniture for $37,000. At the time of the sale, the taxpayer’s depreciation deductions totaled $20,700. What part of the gain is taxed as long-term capital gain?

a.

$7,000

b.

$27,700

c.

$20,700

d.

$0

A

Choice “a” is correct. When property is sold, the realized gain is the difference between the proceeds and the adjusted basis, in this question, the difference between the $37,000 and $9,300 ($30,000 - $20,700), or $27,700. Under Section 1245 (office furniture qualifies as Section 1245 property because it is not real property), the total depreciation deducted will be recaptured as ordinary income, and the remainder (any amount in excess of the original cost) will be Section 1231 (taxed as a long-term capital) gain. In this question, the $20,700 of depreciation deductions is ordinary income and $7,000 ($37,000 - $30,000) is Section 1231 (taxed as a long-term capital) gain.

[Note: Section 1245 actually requires that the lesser of the depreciation taken ($20,700) or the gain recognized ($37,000 - an assumed $9,300 = $20,700) be recaptured. Normally, the two steps of the formula produce the same result.]

Choice “d” is incorrect. The $0 indicates that nothing will be long-term capital gain. Any amount of the proceeds over the original cost (in this question $7,000) will be Section 1231 (taxed as a long-term capital) gain.

Choice “c” is incorrect. The $20,700 is the total depreciation deduction. Normally, that would be the depreciation recaptured. However, in this question, the office furniture, for some reason, is sold for more than the original cost, and there will be Section 1231 gain.

Choice “b” is incorrect. The $27,700 is the entire gain recognized on the transaction. For that amount to be Section 1231 (taxed as a long-term capital) gain, depreciation recapture would have to be ignored.